BẤT ĐẲNG THỨC TRONG SỐ HỌC VÀ MỘT SỐ DẠNG TOÁN LIÊN QUAN

80 15 0
BẤT ĐẲNG THỨC TRONG SỐ HỌC VÀ MỘT SỐ DẠNG TOÁN LIÊN QUAN

Đang tải... (xem toàn văn)

Tài liệu hạn chế xem trước, để xem đầy đủ mời bạn chọn Tải xuống

Thông tin tài liệu

a) Gọi p là ước số nguyên tố bất kì của số phi chính phương c với số mũ là s. Nếu số b không là số phi chính phương thì nó phải chia hết cho.. b) Nếu số chính phương chia hết cho số nguy[r]

(1)

TRƯỜNG ĐẠI HỌC KHOA HỌC

LÊ THỊ HỒNG THÚY

BẤT ĐẲNG THỨC

TRONG SỐ HỌC VÀ MỘT SỐ DẠNG TOÁN LIÊN QUAN

LUẬN VĂN THẠC SĨ TOÁN HỌC

(2)

TRƯỜNG ĐẠI HỌC KHOA HỌC

LÊ THỊ HỒNG THÚY

BẤT ĐẲNG THỨC

TRONG SỐ HỌC VÀ MỘT SỐ DẠNG TOÁN LIÊN QUAN

Chuyên ngành: PHƯƠNG PHÁP TOÁN SƠ CẤP Mã số: 46 01 13

LUẬN VĂN THẠC SĨ TOÁN HỌC

Người hướng dẫn khoa học: GS.TSKH Nguyễn Văn Mậu

(3)

Mục lục

MỞ ĐẦU ii

Chương Các tính toán tập hữu hạn số nguyên 1.1 Số nguyên tính chất liên quan 1.2 Một số đồng thức số học 1.2.1 Một số đẳng thức hàm d(n), σ(n) ϕ(n) 1.2.2 Đẳng thức tổng bình phương 10 1.2.3 Biểu diễn số tự nhiên thành tổng lập phương 15

Chương Bất đẳng thức số học 28

2.1 Bất đẳng thức tập số nguyên 28 2.2 Bất đẳng thức lớp hàm số học 32

Chương Một số dạng toán liên quan 60

3.1 Các dạng toán bất đẳng thức số học qua kỳ Olympic 60 3.2 Các đề toán toán rời rạc liên quan 64 3.2.1 Một số toán cực trị tập số nguyên 64 3.2.2 Một số toán sử dụng phương pháp suy luận 68

KẾT LUẬN 74

(4)

MỞ ĐẦU

Chuyên đề số học nội dung quan trọng bậc trung học phổ thông

Các dạng toán đếm số phần tử, so sánh thứ tự số tập hợp

là nội dung đề thi HSG quốc gia Olympic toán khu vực

quốc tế

Đặc biệt lý thuyết số, hàm số học liên quan đến tính tốn

ước số nguyên, gắn với phép đếm số ước số dạng toán liên

quan đến biểu diễn số nguyên trọng tâm khảo sát đẳng thức

bất đẳng thức số học

Luận văn nhằm mục đích tìm hiểu chi tiết tính chất hàm số học

và số dạng toán bất đẳng thức cực trị liên quan số học

Ngoài phần Mở đầu Kết luận, luận văn chia thành ba chương đề cập

đến vấn đề sau đây:

Chương trình bày tốn đếm, ước lượng thứ tự

Chương trình bày dạng bất đẳng thức tính toán liên quan đến

tập rời rạc hàm số học

Chương trình bày số toán cực trị đề thi học sinh giỏi

quốc gia, Olympic khu vực quốc tế liên quan đến bất đẳng thức số học

Luận văn hoàn thành hướng dẫn khoa học Nhà giáo nhân

dân, GS.TSKH Nguyễn Văn Mậu Tác giả xin bày tỏ lòng biết ơn chân

thành sâu sắc tới GS - Người thầy nghiêm khắc, tận tâm công việc

và truyền thụ nhiều kiến thức quý báu kinh nghiệm nghiên cứu

khoa học cho tác giả suốt trình học tập, nghiên cứu đề tài

Tác giả xin bày tỏ lòng biết ơn chân thành đến Ban Giám hiệu, Phòng

(5)

Thái Nguyên, thầy cô giáo tham giảng dạy hướng dẫn khoa học

cho lớp Cao học toán K10C

Tác giả xin chân thành cảm ơn Ban Giám hiệu, tập thể giáo viên tốn trường

THPT Lý Nhân Tơng, thành phố Bắc Ninh gia đình tạo điều kiện cho tác

(6)

Chương Các tính tốn tập

hữu hạn số nguyên

1.1 Số nguyên tính chất liên quan

Trước tiên, ta xét số hàm số học

Định nghĩa 1.1 (Hàm số Euler ϕ(n)) Cho số tự nhiên n ≥ Ta ký hiệu ϕ(n) số số tự nhiên bé n nguyên tố với n Quy ước ϕ(1) = Định lý 1.1 Hàm ϕ(n) có tính chất nhân tính theo nghĩa: Nếu a, b hai số nguyên tố

ϕ(ab) = ϕ(a)ϕ(b) Chứng minh

Rõ ràng ta giải thiết a > 1, b > Các số nguyên dương không vượt ab liệt kê sau:

1 a

a + a + 2a

2a + 2a + 3a

ka + ka + (k + 1)a (b − 1)a + (b − 1)a + ba

(7)

Các số cột

y, a + y, 2a + y, , (b − 1)a + y

Giả sử rx số dư chia ax + y cho b Như (ax + y, b) = (rx, b) Dễ dàng

thấy (a, b) = nên rx1 6= rx2 với x1 6= x2 Như ta có đẳng thức tập

hợp

{r0, r1, , rb−1} = {0, 1, , b − 1}

Vậy số x mà (ax + y, b) = số x mà (rx, b) = tức ϕ(b)

Vậy có ϕ(a)ϕ(b) số nguyên tố với a nguyên tố với b Đó số ngun tố với ab Nói cách khác ϕ(ab) = ϕ(a)ϕ(b)

Từ định lý ta suy cơng thức tính ϕ(n) sau Định lý 1.2 Giả sử n = pα1

1 p αk

k phân tích tiêu chuẩn n > Khi

ϕ(n) = n 

1 − p1

 −

p2



 −

pk 

Chứng minh

Theo định lý 1.1, ta có ϕ(n) = ϕ(pα1

1 ) ϕ(p αk

k )

Định lý chứng minh ta chứng tỏ ứng với plà số nguyên tố ϕ(pα) = pα(1 −

p) Thật vậy, p nguyên tố nên với k ≤ p

α thì

(k, p) = k p

Số số k ≤ pα bội p hp

α

p i

= pα−1 Vậy

ϕ(pα) = pα− pα−1= pα(1 − p) Ví dụ 1.1 Với n = 360 = 23.32.5

ϕ(360) = 360 

1 −

 −1

3 

1 −1

 = 96

Tầm quan trọng hàmϕ(n)trong số học thể định lý Euler Sau suy rộng định lý Euler

Định lý 1.3 (Định lý Euler mở rộng) Cho a m hai số tự nhiên Khi ta có

(8)

Chứng minh

Ta phải chứng minh

A = am− am−ϕ(m) = am−ϕ(m)(aϕ(m)− 1) chia hết cho m

Giả sử m có phân tích tiêu chuẩn m = qα1

1 q α2

2 q αk

k

Ta chứng minh (a, qi) = (aϕ(m) − 1) qαi, cịn a q

am−ϕ(m) qαi, vậy A .m.

Thật vậy, (a, qi) = theo định lý Euler

(aϕ(qαii )− 1) qαi

i

Mặt khác,

ϕ(qαi

i ) = q αi

i (1 −

1 qi

) ước ϕ(m) (suy từ cơng thức tính ϕ(m))

Do

(aϕ(m)− 1) (aϕ(qiαi)− 1) qαi.

Nếu a qi

am−ϕ(m) qm−ϕ(m)

Mặt khác, rõ ràng m − ϕ(m) ≥ αi (vì có nhấtαi số không nguyên tố với m

là qα1

1 q α2

2 q αi

i ) Do

am−ϕ(m) qm−ϕ(m) qαi

i

Định lý chứng minh

Định lý 1.4 (Định lý Fermat) Choplà số nguyên tố alà số nguyên không chia hết cho P ta có

ap−1 ≡ (mod p)

(9)

Định lý 1.5 (Định lý Fermat dạng khác) Cho p số nguyên tố a số nguyên tùy ý ta có

ap ≡ a (mod p)

Chứng minh Nếuachia hết chopthì hiển nhiênap≡ a (mod p) Nếua khơng chia hết cho p theo định lý 1.4 ta có ap−1 ≡ (mod p) sau nhân hai vế đồng dư thức với a ta ap ≡ a (mod p)

Ngược lại từ định lý 1.5 ta suy định lý 1.4 Thật vậy, từ ap ≡ a (mod p)vàalà số nguyên không chia hết cho số nguyên tố pthế thìanguyên tố với p nên cách chia cho a ta đượcap−1 ≡ (mod p) Chính vậy, người ta nói định lý 1.5 dạng khác định lý Fermat

Ví dụ 1.2 Tìm số ngun x để 9x + tích hai số nguyên liên tiếp Lời giải Giả sử 9x + = n(n + 1) với n ∈Z⇒ 36x + 20 = 4n2+ 4n suy

36x + 21 = (2n + 1)2 ⇒ 3(12x + 7) = (2n + 1)2

Số phương (2n + 1)2 chia hết chia hết cho Mặt khác (12x + 7) không chia hết 3(12x + 7) không chia hết cho

Mâu thuẫn chứng tỏ không tồn số nguyên x để 9x + = n(n + 1) Ví dụ 1.3 Tìm số nguyên x để biểu thức sau số phương x4+ 2x3+ 2x2+ x +

Lời giải Đặt x4+ 2x3+ 2x2+ x + = y2 với y ∈N.

Ta thấy y2 = (x4+ 2x3+ x2) + (x2+ x + 3) = (x2+ x)2+ (x2+ x + 3)

Đặt x2 + x = a ta có y2 = a2 + (x2 + x + 3) Từ có y2 − a2 = x2 + x + =

 x +

2 2

+11

4 > ⇒ y

2 > a2.

Dễ thấy (a + 2)2− y2= 3x +

2 2

+

40 ⇒ (a + 2)

2 > y2.

Do a2< y2 < (a + 2)2 ⇒ y2= (a + 1)2.

Suy rax4+ 2x3+ 2x2+ x + = (x2+ x + 1)2.Suy rax2+ x − = ⇒ x = 1; x = −2 Thử lại, với x = 1; x = −2 biểu thức = 32 Vậy x = 1; x = −2 giá trị cần tìm

Ví dụ 1.4 Tìm nghiệm ngun dương phương trình

(10)

Lời giải Giả sử x0, y0, z0 thỏa mãn (1.1) có ƯSCLN d

Giả sử x0= dx1, y0 = dy1, z0= dz1 (x1, y1, z1) thỏa mãn (1.1)

Do đó, ta giả sử (x, y, z) = x, y, z đơi ngun tố hai ba số x, y, z có ước chung d số cịn lại chia hết cho d Ta có x.y = z2 mà (x, y) = nên x = a2, y = b2 với a, b ∈N∗ Bởi

(1.1)⇔ z2 = x.y = (ab)2 ⇔ z = (ab).

Như ta biểu thức nghiệm

x = ta2; y = tb2; z = ab (t ∈N∗)

Ngược lại, dễ thấy số x, y, z có dạng thỏa mãn (1.1) Vậy cơng thức cho ta tất nghiệm nguyên dương (1.1)

Ví dụ 1.5 Tìm tất nghiệm ngun phương trình

x2+ xy + y2= x2y2 (1.2) Lời giải (1.2)⇔ x2+ 2xy + y2= x2y2+ xy ⇔ (x + y)2= xy(xy + 1).

Ta thấy xy xy + hai số nguyên liên tiếp nên: + Xét xy = 0, ta có xy = x2+ y2=

⇔ x = y = + Xét xy + = 0, ta có : xy = −1và x2+ y2=

⇔ (x, y) = (1, −1); (−1, 1)

Thử lại, ba cặp số (0, 0); (1, −1); (−1, 1) thỏa mãn phương trình cho Vậy phương trình có ba nghiệm nguyên (x, y) = (0, 0); (1, −1); (−1, 1)

b Hàm tổng ước số tự nhiên

Định nghĩa 1.2 (xem [2],[3]) Cho số nguyên dương n Ta ký hiệu σ(n)là tổng ước n

(11)

Chứng minh

Thật vậy, giả sử a1, ak ước a, k = d(a) b1, , bl ước

b, l = d(b) Khi aibj(1 ≤ i ≤ k, ≤ j ≤ l) tất ước ab Vậy

σ(ab) = l P j=1 k P i=1

aibj =

 k P i=1  l P j=1 bj 

= σ(a)σ(b)

Từ định lý ta suy cơng thức tính σ(n) sau: Định lý 1.7 (xem [2],[3]) Giả sử n = pα1

1 p α2

2 p αk

k phân tích tiêu chuẩn

n Khi σ(n) = p

α1+1

1

p1−

pα2+1

2

p2−



.p

αk+1

k

pk −



Chứng minh

Theo định lý , ta có σ(n) = σ(pα1

1 )σ(p α2

2 ) σ(p αk

k )

Vậy ta cần chứng minh σ(pα) = p

α1+1

1 −

p1−

Thật vậy, tất ước pα 1, p, p2, , pα Do σ(pα) = + p + · · · + pα = p

α+1−1

p −

Định lý 1.8 (xem [2],[3]) a) n số nguyên tố σ(n) = n + b) σ(n) số lẻ n số phương n

2 số phương

Chứng minh

a) Nếu n số nguyên tố σ(n) = n + Ngược lại, σ(n) = n + n hợp số n có ước 1, a n với (1 < a < n) Vậy σ(n) ≥ n + + a > n + Nếu n = σ(n) = < n +

b) Nếu n = m2 n = 2m2 n = 2αpα1

1 p α2

2 p αk

k , p1, p2, , pk số

nguyên tố lẻ αi chẵn Khi σ(n) = σ(2α)σ(pα11)σ(pα22) σ(pαkk)

Ta có σ(2α) = 2α+1− số lẻ, σ(pαi

i ) = + pi+ · · · + p αi

i số lẻ tổng

của số lẻ số lẻ Vậy σ(n) lẻ Đảo lại, giả sử σ(n) lẻ, giả sử n = 2αpα1

1 p α2

2 p αk

k Khi với i, σ(p αi

i )

số lẻ Mặt khác σ(pαi

i ) = + pi+ · · · + p αi

i ≡ α1+ (mod 2) suy αi chẵn Do

đó, α chẵn n số phương Nếuα lẻ n

2 số phương Liên quan đến hàm σ(n) ta có khái niệm số hồn chỉnh

(12)

σ(n) = 2n, tức n tổng ước (khơng kể nó)

Ví dụ 1.6 6, 28 số hồn chỉnh, = + + 3, 28 = + + + + 14 Ngay từ thời cổ Hi Lạp, nhà toán học Euclid chứng minh kiện lý

thú sau:

Định lý 1.9 (xem [2],[3]) Nếu k số tự nhiên cho 2k− số nguyên tố số n = 2k−1(2k − 1) số hoàn chỉnh

Chứng minh

Đặt p = 2k− Ta có σ(n) = σ(2k−1)σ(p) = (2k− 1)(p + 1) = (2k − 1).2k = 2n.

Rõ ràng số hồn chỉnh có dạng số chẵn (vì k > 1)

Đến kỷ 18, Euler phát rằng: Tất số hồn chỉnh chẵn

có dạng Ta có định lý sau:

Định lý 1.10 (xem [2],[3]) Nếu n số hồn chỉnh chẵn n có dạng n = 2k(2k+1− 1)

Ở k ≥ 2k+1− số nguyên tố

Chứng minh

Ta biểu diễn n dạng n = 2kb với k ≥ b số lẻ Ta có σ(n) = σ(2k)σ(b) = (2k+1− 1)σ(b)

Mặt khác, n số hồn chỉnh σ(n) = 2n = 2k+1b Vậy suy (2k+1− 1)σ(b) = 2k+1b. Hay b

σ(b) =

2k+1− 2k+1

Vì 2k+1 2k+1− nguyên tố nên tồn cđể b = (2k+1− 1)c, σ(b) = 2k+1c

Vì k ≥ nên b > c tức ước b Nếu c > b, c, 1là ước b, σ(b) ≥ b + c + = (2k+1− 1)c + c + = 2k+1c + 1, điều trái với σ(b) = 2k+1c Vậy c = Do b = 2k+1− n = 2kb = 2k(2k+1− 1)

Vì σ(b) = 2k+1 = b + nên b số nguyên tố

(13)

nguyên tố k phải số nguyên tố Một số nguyên tố dạng 2k − gọi số nguyên tố Mersenne Cho đến năm 2018 người ta tìm 50 số nguyên

tố k để 2k − số nguyên tố Đó số 2, 3, 5, 7, 13, 17, 19, 31, 61, 89, 107, 127, 521, 607, 1279, 2203, 2281, 3217, 4253, 4423, 9689, 9941, 11213, 19937,

21701, 44497, 86243, 23209, 110503, 132049, 216091, Và số k thứ 49 tìm

được năm 2016 74207281 cịn đến năm 2018 kỷ lục tìm ứng với k =

77232917

Người ta chưa biết tập hợp số nguyên tố Mersenne hữu hạn hay vơ

hạn chưa biết tập hợp số hoàn chỉnh chẵn hữu hạn hay vô

hạn

Cho đến người ta chưa tìm thấy số hồn chỉnh lẻ khơng

biết liệu có số hồn chỉnh lẻ hay khơng Có giả thuyết cho khơng có số

hoàn chỉnh lẻ

1.2 Một số đồng thức số học

1.2.1 Một số đẳng thức hàm d(n), σ(n) ϕ(n)

Bài toán 1.1 (xem [1],[3]) Cho dãy số nguyên dương xn xác định theo quy

luật sau x0= a, xn+1 = d(xn), (∀n ≥ 0)

a) Chứng minh ∀a ∈N∗ tồn số n0 (phụ thuộc a) cho xn = với

mọi n ≥ n0

b) Hãy cho ví dụ chứng tỏ số n0 lớn tùy ý a đủ lớn

Lời giải Ta có d(n) ≤ 2√n < nnếun > Với n = n = thử trực tiếp cho thấy d(4) = < 4, d(3) = <

Với n = 2, d(2) =

Vậy ta có d(n) < n, ∀n > d(2) = Suy xn+1< xn xn >

Vì (xn) số nguyên dương nên số n0 ta phải có

(14)

b) Lưu ý d(2n−1) = n Xét dãy mk xác định m0 = 3, mk+1 = 2mk−1, ta

có d(mk+1) = mk Vậy d d(mk+1)

| {z }

k+1

= m0= Nghĩa là:

Nếu a = mk xn = n ≥ k + lớn tùy ý k lớn tùy ý

Bài toán 1.2 (xem [1],[3]) Chứng minh σ(n) = 2n + n bình phương số lẻ

Lời giải Vì σ(n) = 2n + số lẻ theo định lý 6.4 ta có n = 2αm2, α ≥ m số lẻ Ta cần chứng minh α =

Ta có σ(n) = 2n + = 2α+1m2+ Do tính chất nhân tính hàm σ(n) ta lại có σ(n) = σ(2α).σ(m2) = (2α+1− 1).σ(m2)

Vậy 2α+1m2+ = (2α+1− 1).σ(m2) 2α+1−

Ta lại có 2α+1m2+ = 2α+1m2+ − m2+ m2 = m2(2α+1− 1) + (m2+ 1) .2α+1− 1.

Suy m2+ 2α+1−

Nếu a > 2α+1− có dạng 4k − 1, có ước nguyên tố p dạng 4k − Vậy m2 ≡ −1 (mod p)

Suy mp−1≡ (−1)p−12 = (−1) (mod p)

Điều trái với định lý Fermat Vậy α =

Bài toán 1.3 (xem [1],[3]) Chứng minh ϕ(n) ≥ √

n

2 với n, ϕ(n) < n −√n n hợp số

Lời giải Giả sử n có phân tích tiêu chuẩnn = 2αpα1

1 p α2

2 p αk

k Theo cơng thức

tính ϕ(n) ta có ϕ(n) = 2α−1pα1−1

1 p

α2−1

2 p αk−1

k (p1− 1) (pk− 1)

Chú ý pi− ≥

pi αi−

1 ≥

αi

2 ta có

ϕ(n) ≥ 2α−1pα1

1 −

1 p

αk

k −

1 ≥

α−1.p

α1

2

1 p

αk k ≥ √ n Giả sử n hợp số Gọi p1 ước nguyên tố bé n Khi đóp1 ≤

√ n Do ϕ(n) ≤ n(1 −

p1

) = n − n p1

≤ n −√n

(15)

nhất số nguyên dương n ϕ(n + k) = ϕ(n)

Lời giải Nếu k lẻ ϕ(k + k) = ϕ(2k) = ϕ(2)ϕ(k) = ϕ(k) Vậy chọn n = k Xét trường hợp k chẵn Gọi p số nguyên tố bé tập hợp số nguyên tố ước k Ta có ϕ(pk) = ϕ(p)ϕ(k) = (p − 1)ϕ(k) Giả sử p1, p2, , pr ước nguyên tố k Mọi ước nguyên tố p −

nằm tập (p1, p2, , pr) cách chọn p Vậy thì:

ϕ((p − 1)k) = (p − 1)k(1 − p1

) (1 − pr

) = (p − 1)ϕ(k)

Suy ϕ(pk) = ϕ((p − 1)k) Vậy chọn n = (p − 1)k

1.2.2 Đẳng thức tổng bình phương

Định lý 1.11 (Tổng hai bình phương) Giả sử nđược biểu diễn dạng phân tích chuẩn

n = 2rΠpsi

i q tj

j , pi≡ (mod 4), qj ≡ (mod 4)

Điều kiện cần đủ để n biểu diễn thành tổng hai bình phương số tj

chẵn với j

Để chứng minh định lý ta cần sử dụng bổ đề sau:

Bổ đề 1.1 Giả sử số nguyên tố q \ a2+ b2 Nếu q ≡ (mod 4) q \ a, q \ b

Chứng minh Dễ thấy q \ a q \ b Giả sử ngược lại q -a, q - b Khi theo giả thiết ta có a2+ b2 ≡ (mod q) aq−1 ≡ (−1)q−12 bq−1 (mod q)

Theo định lý Fermat, ta có(−1)q−12 ≡ (mod q)hay (-1) số phương theo

mod q nên q = 4k + Mâu thuẩn chứng minh bổ đề

Bổ đề 1.2 Tích hai số mà số tổng hai bình phương hai số

ngun khơng âm tổng bình phương hai số khơng âm

(16)

Bổ đề 1.3 Mọi số nguyên tố p dạng 4k + biểu diễn thành tổng bình phương hai số nguyên dương

Chứng minh Giải sử p = 4k + Xét a = (2k)! Ta có (2k)! = (−1)2k(2k)! = (−1)(−2) (−2k) ≡ (p − 1)(p − 2) (p − 2k)

= 4k(4k − 1) (2k + 1) (mod p)

Do đóa2 ≡ (2k)!(2k +1) 4k = (4k)! = (p−1)! ≡ −1 (mod p)theo định lý Wilson Ký hiệuq = [√p] Xét(q +1)2các số dạng ax+yvới x, y = 0, 1, , q Vì(q +1)2pq2 nên theo Dirichlet tồn cặp số (x1, y1), (x2, y2) cho ax1+ y1 ≡ ax2+ y2

(mod p) hay a(x1− x2) + (y1− y2) chia hết cho p

Đặt x = |x1 − x2|, y = |y1− y2| Ta có a2x2− y2 = (ax − y)(ax + y) Theo

a2 ≡ −1 (mod p) Vậy x2+ y2≡ −a2x2+ y2 ≡ (mod p)

Mặt khác x2 ≤ q2hp, y2 ≤ q2 < p mà p nguyên tố nên < x2+ y2 < 2p Suy x2+ y2 = p Rõ ràng x 6= 0, y 6=

Bổ đề chứng minh

Chứng minh định lý 1.11

Điều kiện đủ: Giả sử tj chẵn ∀j Ta có = 12+ 12 số nguyên tố p có

dạng4k + biểu diễn thành tổng bình phương hai số nguyên dương, theo bổ để 1.3

Theo bổ đề 1.2, ta có m = 2rΠpsi

i = a

2+ b2, a, b ∈

Z Mặt khác, tj chẵn ∀j nên

Πqtj

j = h2 n = m.h2= a2+ b2h2= (ah)2+ (bh)2

Điều kiện cần: Giải sử n = a2+ b2, a, b ∈ Z ta chứng minh các tj chẵn ∀j

bằng phương pháp phản chứng Giải sử ∃qj nguyên tố dạng 4k + ước n

có số mũ tj lẻ Khi n = qjtjB, (B, qj) = Từ a2+ b2 qjtj qj Theo bổ đề 1.1

ta có a = a1qj, b = b1qj Do a21+ b21 = q tj−2

j B

Nếu tji2, tiếp tục sau hữu hạn bước ta có a2k+ b2k = qiB tj lẻ

ak = qjak+1, bk = qjbk+1 Suy qj(a2k+1+ b2k+1) = B dẫn đến B qj

Mâu thuẫn chứng minh định lý

(17)

- Phương trìnhx2+ y2 = 20032003 vơ nghiệm số ngun tố 2003 có dạng 4k + số mũ 20032003 lẻ

Định lý 1.12 (Tổng ba bình phương) Các số có dạng 4n(8k + 7) khơng thể biểu diễn thành tổng ba bình phương

Chứng minh Giả sử 4n(8k + 7) = x2+ y2+ z2 với x > 0, y ≥ 0, z ≥ Do a2 có dạng4k 4k + nên x2+ y2+ z2 có dạng4k x, y, z chẵn Đặt x = 2x1,y = 2y1, z = 2z1, đẳng thức có dạng

4n−1(8k + 7) = x21+ y12+ z12, x1> 0, y1 ≥ 0, z1≥

Tiếp tục lập luận n lần, ta có (∗)8k + = x2n+ yn2 + zn2, xn > 0, yn ≥

0, zn ≥ Mặt khác a2 có dạng 8k, 8k + 8k + nên

Nếu yn = zn = hay y = z = (*) không xảy

Nếu yn 6= 0, zn = hay y 6= 0, z = (*) không xảy

Nếu xn, yn, zn khác khơng với trường hợp sau (*) không xảy

i Một ba số lẻ, hai số lại chẵn

ii Hai ba số lẻ

iii Ba số lẻ

iv Ba số chẵn

Vậy định lý chứng minh

Chú ý 1.1 Gauss chứng minh số dạng 4n(8k + 7) biểu diễn thành tổng ba bình phương

Định lý 1.13 (Định lý Lagrange tổng bốn bình phương) Một số nguyên

dương biểu diễn thành tổng bốn bình phương số ngun

khơng âm

Trước hết ta sử dụng bổ đề sau:

Bổ đề 1.4 Tích hai số nguyên dương mà số tổng bốn bình

phương số nguyên không âm tổng bốn bình phương số

ngun khơng âm

(18)

= (x1y1+ x2y2+ x3y3+ x4y4)2+ (x1y1− x2y1+ x3y4− x4y3)2+ (x1y3− x3y1+ x4y2−

x2y4)2+ (x1y4− x4y1+ x2y3− x3y2)2

Với xi, yi số nguyên không âm, i = 1, 2, 3,

Bổ đề 1.5 Nếu p số nguyên tố lẻ tồn k, < k < p cho kp tổng bốn bình phương số ngun khơng âm

Chứng minh Xét

2(p+1)số x

2,x = 0, 1, 2, ,p −

2

2(p+1)số−y

2− 1, y =

0, 1, 2, ,p −

2 Các số tập đôi phân biệt theo mod p hai tập có (p + 1) số phân biệt theo mod p Theo nguyên lý Dirichlet, tồn x, y ∈ 0, 1, 2, ,p −

2 chox

2 ≡ −y2− (mod p) suy ra x2+ y2+ = kp ⇒

kp = x2+ y2+ 12+ 02

Mặt khác kp = x2+ y2+ 12 < p

2

4 + = p2

2 + < p

2 ⇒ k < p

Vậy bổ đề chứng minh

Bổ đề 1.6 Nếu plà số nguyên tố pđược biểu diễn thành tổng bốn bình phương số ngun khơng âm

Chứng minh Từ = 12+ 12+ 02+ 02 nên ta cần xét p ≥ Theo bổ đề 1.5 ta có kp = x21+ x22+ x23+ x24, < k < p

Gọi k0p bội nhỏ thỏa mãn điều kiện Ta chứng minhk0= Giả

sử k0i1, ta thấy k0 lẻ Thật vậy, k0 chẵn x1+ x2+ x3+ x4 chẵn

do x1, x2, x3, x4 chẵn lẻ hai bốn số chẵn, hai

số lẻ, giả sử x1, x2 chẵn x3, x4 lẻ Khi ba trường hợp cho kết

x1+ x2, x1− x2, x3+ x4, x3− x4 chẵn

1

2k0p = (

x1+ x2

2 )

2+ (x1− x2

2 )

2+

(x3+ x4 )

2+ (x3− x2

2 )

2. Điều trái với định nghĩa của k

0 Do k0 phải lẻ

Lúc đók0 lẻk0 ≥ 3vàk0khơng ước đồng thời xi, i = 1, 2, 3, 4(vìk0 -p)

Ta chọn b1, b2, b3, b4 cho yi= xi− bik0, |yi| <

k0

2 y

2

1+ y22+ y23+ y42>

Do 0hy12+ y22+ y32+ y42h4(k0 )

2 = k2

0, y21+ y22+ y23+ y42≡ (mod k0)

Như

x21+ x22+ x32+ x24 = k0p, < k1 < p

(19)

Theo bổ đề 1.4 ta k02k1p = z21+ z22+ z32+ z42, z1, z2, z3, z4 xác

định

z1 = x1y1+ x2y2+ x3y3+ x4y4 ≡ x21+ x22+ x23+ x24 ≡ (mod k0)

Tương tự z2 ≡ (mod k0), z3≡ (mod k0), z4≡ (mod k0)

Từ zi= k0t(i = 1, 2, 3, 4) ta có

k1p = t21+ t22+ t23+ t24

Điều mâu thuẫn với định nghĩa k0 Vậy k0 = Bổ đề chứng

minh

Ví dụ 1.8 2.7 = 32+ 22+ 12+ 02, x1 = 3, x2 = 1, x3 = 2, x4= Do

3 +

2

+3 −

2

+2 +

2

+2 −

2

= 22+ 12+ 12+ 12 = 1.7 =

Chứng minh định lý 1.13 Đầu tiên ta có = 12+ 02+ 02+ 02 Giả sử n ≥ n phân tích thành tích số nguyên tố Theo bổ đề 1.6 1.4 ta n tổng bốn bình phương số ngun khơng âm

Ví dụ 1.9 55 = 5.11

5 = 22+ 12+ 02+ 02 11 = 32+ 12+ 12+ 02 Do đó55 =



6 + + + 2

+ 

2 − + + 2

+ 

2 − − + 2

+ 

0 + − + 2

= 72+ 12+ 22+ 12

Định lý 1.14 (Tổng năm bình phương) Mỗi số ngun dương ni169 ln biểu diễn thành tổng năm bình phương số nguyên dương

Chứng minh Giả sử ni169, theo định lý 1.13, ta có n − 169 = x21+ x22+ x23+ x24, x1≥ x2 ≥ x3≥ x4 ≥

Nếu xii0(i = 1, 2, 3, 4) n = 132+ x12+ x22+ x23+ x24

Nếu x1, x2, x3i0, x4= n = 122+ 52+ x21+ x22+ x23

Nếu x1, x2i0, x3 = x4 = n = 122+ 42+ 32+ x21+ x22

Nếu x1i0, x2= x3 = x4 = n = 102+ 82+ 22+ 12+ x21

(20)

1.2.3 Biểu diễn số tự nhiên thành tổng lập phương

Bài toán 1.5 (Bài toán Waring) Xét toán biểu diễn số thành tổng

các lũy thừa k Vào kỷ 18 nhà toán học Anh Waring đoán rằng, số nguyên dương biểu diễn thành tổng lập phương số

tự nhiên biểu diễn thành tổng 19 lũy thừa số tự nhiên,

tức với n ∈ N∗ phương trình

x31+ x32+ · · · + x39= n x41+ x42+ · · · + x419 = n ln có nghiệm tự nhiên Ơng nêu giả thiết sau:

Cho số nguyên dương k Khi có tồn số nguyên dương m (phụ thuộc vào k) cho số nguyên dươngn biểu diễn thành tổng củam số, số có dạng xk, x ∈N tức là:

Tồn số nguyên dương m cho với số nguyên dương n phương trình

m

X

i=1

xki = n

có nghiệm tự nhiên

Năm 1906 nhà toán học lỗi lạc Davit Hilbert chứng minh

đốn Chứng minh ơng phức tạp

Gọi g(k) số m nhỏ có tính chất trên, tức số ngun dương n biểu diễn thành tổng g(k) số, số có dạng xk, x ∈N tồn số n không biểu diễn thành tổng m − số, số có dạng xk, x ∈N Ta có g(2) = (vì số khơng biểu diễn thành tổng bình phương số nguyên dương n biểu diễn thành tổng bốn bình phương) Người ta chứng minh g(3) = 9, g(4) ≥ 19, g(5) = 37 với k ≤ 471600000

g(k) = [(3 2)

k] + 2k− 2.

Vẫn nhiều câu hỏi mở xung quanh hàm g(k)

Các toán số lũy thừa, nói riêng số phương, thường khơng cần

nhiều vốn kiến thức, địi hỏi phân tích tổng hợp giả thiết cách

(21)

luận đầy đủ Chính mà toán số lũy thừa thường gặp

kì thi chọn học sinh giỏi cấp 2, cấp 3, thi toán quốc tế thi tuyển vào

lớp 10

Các toán số lũy thừa phong phú, trình bày số kiến

thức dùng để xét xem số có số phương, số lũy thừa hay

không; đồng thời nêu số toán liên quan đến dạng số lũy thừa

Cịn nhiều tốn số lũy thừa hệ thập phân chưa nêu

Định nghĩa 1.4 (Số lũy thừa) a) Ta gọi lũy thừa bậc r(r ≥ 2) số tự nhiên a, tức số ar, số lũy thừa

b) Ta gọi bình phương số tự nhiên a, tức số a2, số phương, số phương số lũy thừa bậc hai

c) Số nguyên dương không chia hết cho số phương lớn gọi

là số phi phương

Chẳng hạn, số sau số phi phương: 3; 5; 7; = 2.3; 30 = 2.3.5 Các

số sau khơng số phương khơng số phi phương:

12 = 22.3; 60 = 22.3.5

Chú ý 1.2 1) Số 0, số số phương số lũy thừa bậc tùy ý

2) Các tên gọi số lũy thừa, số phương, số phi phương sử dụng

cho số ngun khơng âm

Tính chất 1.1

a) Số phi phương số nguyên tố lớn 1, tích số

nguyên tố phân biệt với số mũ

b) Mỗi số nguyên dươnga biểu diễn dạng tích số phương số phi phương, tức có dạng a = b2c

Chứng minh

a) Gọi p ước số nguyên tố số phi phương c với số mũ s Nếu s ≥ c chia hết cho p2, trái với định nghĩa số phi phương, s =

(22)

một số phương k2 > 1, lúc b = k2n nên a = c2b = c2k2n = (ck)2n mà ck > c, trái với xác định số c

Giả sử số a có hai cách biểu diễn a = c2b = e2f, b, f số phi phương Đặt (c, e) = d c = dc1, e = de1 (c1, e1) = Lúc c2b = e2f ⇔

d2c21b = d2e12f ⇔ c21b = e21f Từ (c1, e1) = (c21, e21) = nên e21 ước số

của b Do b số phi phương e1 = Xét tương tự c1 = Vậy cách

biểu diễn a = c2b Tính chất 1.2

a) Nếu số lũy thừa bậc n chia hết cho số nguyên tố p số chia hết cho pn b) Nếu số phương chia hết cho số ngun tố p số chia hết cho p2 Chứng minh

a) Giả sử cn chia hết cho số nguyên tố p với n ≥ Nếu (c, p) = (cn, p) = 1, điều không xảy nên (c, p) = p , tức c chia hết cho p , cn chia hết cho pn Khi n = có câu b)

Tính chất 1.3

a) Nếu số lũy thừa bậc n tích hai số nguyên tố nhau, tức cn = a.b với (a, b) = 1, thừa số a, b số lũy thừa bậc n

b) Nếu số phương tích hai số nguyên tố nhau, tức c2= a.b với (a, b) = 1, thừa số a, b số phương

Chứng minh

a) Đặt (a, c) = e a = ed c = em với (d, m) = Từ cn = a.b với n ≥ có enmn = edb ⇔ en−1mn = db Vì (a, b) = (e, b) = 1, đồng thời có en−1mn = db nên b ước mn Từ (d, m) = (d, mn) = 1, đồng thời có en−1mn = db mn ước b Suy b = mn d = en−1 Từ có a = ed = dn Khi n = có câu b)

Tính chất 1.4 Căn bậc n số nguyên dương số nguyên dương, số vơ tỉ Nói cách khác, an = d với d số nguyên dương mà a số hữu tỉ a số nguyên

Chứng minh Giả sử dn1 = a ⇔ an = d với d số nguyên dương Giả sử

a = r

(23)

ra (s, rn) = Từ điều giả sử có rn = ansn = dsn , suy s ước số rn nên s = (s, rn) = Vậy a số hữu tỉ a = r số nguyên dương

Tính chất 1.5 Giả sử a, b, m, n số nguyên dương thỏa mãn am = bn (m, n) = tồn số nguyên dương c cho a = cn b = cm

Chứng minh Theo điều kiện có nghiệm phương trình vơ định bậc nhất,

nếu (m, n) = tồn số nguyên dương x, y cho mx − ny = Từ cób = bmx−ny = b

mx

bny =

bmx amy =

bx ay

m

Theo tính chất 3.4 b

x

ay = c số nguyên

dương, suy b = cm, thay vào am = bn a = cn

Tính chất 1.6 Cho số nguyên s ≥ chọn số nguyên ns cho với

mỗi số nguyên m ≥ ns tồn số lũy thừa as thỏa mãn m < as < 2m

Chứng minh Giả sử có m < as < 2m m1s < a < (2m)

s Để tồn số a

ngun cần có m1s + < (2m)

s ⇔ (2m) s − m

1

s > ⇔ m s(2

1 s −

1

s) > ⇔ m >

1 (21s −

1 s)s

Ta cần chọn số nguyên ns ≥ +

1 (21s −

1 s)s

thì với m ≥ ns ≥ +

1 (21s −

1 s)s

sẽ

có m1s < a < (2m) s

Tính chất 1.7 Giả sử a, b, n số nguyên dương

a) Nếu số b thỏa mãn an < b < (a + 1)n số b không số lũy thừa bậc n b) Nếu số b thỏa mãn a2 < b < (a + 1)2 số b khơng số phương

Chứng minh

a) Giả sửb = cn cóan < cn < (a + 1)n , suy raa < c < (a + 1), không tồn số nguyên c Khi n = ta thu kết câu b) Định lý 1.15 (Định lí Liouville) Với số nguyên dươnga n ≥ đẳng thức (a − 1)! + = an xảy a =

(24)

Theo giả thiết có (a − 2)!(a − 1) = an− = (a − 1)(an−1+ an−2+ · · · + a + 1) ⇔ (a − 2)! = (an−1− 1) + (an−2− 1) + · · · + (a − 1) + n (1.3) Với a > a − > a −

2 > nên (a − 2)! chứa tích a −

2 = a − Từ (1.3) suy a − ước n, n ≥ a − Thay vào (1.3) (a − 2)! ≥ aa−2+ aa−3+ · · · + a + > aa−2 , điều không xảy nên không tồn số a Vớia ≤ xảy đẳng thức 4! + = 52 Nhận xét 1.1 Ta biết số đẳng thức dạnga!+1 = b2 (n > 1)như :4!+1 = 52 ; 5! + = 112 ; 7! + = 712 Nhà toán học M Kraitchik kiểm tra (năm 1924) với a ≤ 1020 khơng có số a để a! + số phương

Ta khơng biết với a > 1020 có số a để a! + số phương hay khơng?

Định lý 1.16 (xem [2],[3])

a) Số phương có chữ số tận chữ số 0, 1, 4, 5, 6,

và khơng có chữ số tận 2, 3, 7,

b) Nếu số phương có chữ số tận hai chữ số cuối 25

c) Nếu số phương có chữ số tận chữ số hàng chục chữ số

lẻ

d) Nếu số phương có chữ số tận là chữ số lẻ 1, 5, chữ

số hàng chục chữ số chẵn

Chứng minh Xét số n = 10k + r với k, r số nguyên ≤ r ≤ n2 = (10k + r)2 = 20k(5k + r) + r2.Vì r2 00, 01, 04, 09, 16, 25, 36, 49, 64, 81 nên chữ số tận số phương 0, 1, 4, 5, 6,

Hơn nữa, 20k(5k + r) số chẵn nên chữ số hàng chục số phương có tính chẵn lẻ với r2, từ kết luận hai chữ số cuối n2 Định lý 1.17 (xem [2],[3])

a) Số phương chia cho có dạng 3n 3n + khơng có dạng 3n +

(25)

c) Số phương chia cho có dạng 5n, 5n + 1, 5n + 4và khơng có dạng 5n + 2, 5n +

d) Số phương chia cho có dạng 6n, 6n + 1, 6n + 6n + khơng có dạng 6n + 2, 6n +

e) Số phương chia cho có dạng 8n 8n + 8n + khơng có dạng 8n + r với r 2, 3, 5, 6,

g) Số phương chia cho có dạng9n hoặc9n + 1hoặc 9n + 4hoặc9n + khơng có dạng 9n + r với r 2, 3, 5, 6,

Chứng minh Áp dụng lập luận chứng minh định lý 1.16

a) Xét n = 3k + r với k, r số nguyên ≤ r ≤ n2 = (3k + r)2 = 3k(3k + 2r) + r2 r2 0, 1, 4, từ rút kết luận

Chứng minh tương tự cho trường hợp lại

Định lý 1.18 (xem [2],[3])

a) Số lũy thừa bậc ba chia cho khơng có dạng 4n +

b) Số lũy thừa bậc ba chia cho khơng có dạng 8n + r với r 2, 4, c) Số lũy thừa bậc ba chia cho khơng có dạng9n+rvớirbằng 2, 3, 4, 5, 6,

Chứng minh

a) Chứng minh tương tự chứng minh định lý 1.16 đpcm

Để chứng minh số số lũy thừa ta dùng tính chất 1.4, tính

chất 1.5, tính chất 1.6, tính chất 1.7, biến đổi số xét thành lũy thừa

bậc n số ngun

Tiếp theo, ta tính tốn số lũy thừa cho

Bài toán 1.6 (xem [1],[3]) Chứng minh cặp số nguyên dương(m, n)mà tổng tích chúng số phương chúng có dạngm = ka2, n = kb2 , a2+ b2= kc2 với k số phi phương

Lời giải Giả sử m + n = q2 mn = e2 Đặt d = (m, n) m = dm1, n = dn1

(26)

được d2m1n1 = e2 Từ d2 ước e2, suy d ước e, tức e = dh ,

thay vào đẳng thức m1n1 = h2 Từ (m1, n1) = m1 = u2

và n1 = v2 theo tính chất 1.5, tức m = du2, n = dv2 Theo tính chất 1.3

viết d = kg2 với k số phi phương Đặt a = gu, b = gv viết m = kg2u2 = ka2, n = kg2v2 = kb2, k số phi phương q2 = m + n = ka2+ kb2 = k(a2+ b2) Đặt (k, q) = p k = ps, q = pc (s, c) = theo tính chất ước chung lớn Từ đẳng thức k = ps ước q2 = p2c2 , tức p2c2 = tps , suy pc2 = ts , mà (s, c) = nên s ước p, tức p = rs , k = ps = rs2 , k số phi phương nên s = 1, tức k = p q = pc = kc Thay vào đẳng thức q2 = k(a2 + b2) k2c2= q2 = k(a2+ b2) rút kc2 = a2+ b2

Ngược lại, dễ thấy cặp số (m, n) có dạng có tổng tích số phương.(đpcm)

Bài toán 1.7 (xem [1],[3]) Cấp số cộng (nhị thức bậc nhất) chứa số

phương

a) Chứng minh cấp số cộng a + bn (với n = 1, 2, a, b số nguyên, b > 0) chứa số phương cấp số chứa vơ hạn số phương

b) Nếu cấp số cộng a + bn (với n = 1, 2, a, b số nguyên, b > 0) chứa số phương e2 = a + bn tồn số phương r2 = d + bm thuộc cấp số cộng (với m, d số nguyên) thỏa mãn ≤ d ≤ b − −k ≤ r ≤ k, b = 2k b = 2k + 1.Từ cách tìm số phương a + bn c) Tìm số nguyên n cho 27 + 22n số phương

Lời giải

a) Giả sử cấp số cộng a + bn với n = 1, 2, tồn số a + bn0 = e2

các số dạng (bn + e)2 = b2n2+ 2ben + e2 = b(bn2+ 2en + n0) + a số

phương với n = 1, 2,

b) Giả sử với số nguyên a, b, n có số phương a + bn = e2

(27)

Giả sửb = 2k b = 2k + Ta chuyển việc xét số phương a + bn = e2 xét số phương d + bm = r2 với ≤ d ≤ b − −k ≤ r ≤ k (với b = 2k thừa số), dẫn đến tìm số m cho1 − b ≤ −d ≤ bm = r2−d ≤ r2 ≤ k2≤

4b

2,

tức ≤ m ≤ 4b

Như số a + bn = e2 số phương tồn số m cho d + bm = r2 với số d, m, r xác định

c) Ta chuyển việc xét số phương 27 + 22n = e2 xét số phương + 22s = e2 , đón = s − ≥ Đặt e = 22t + r với ˘10 ≤ r ≤ 10thì 22s + = e2 = 22(22t2+ 2tr) + r2 nên 22 ước số củar2− 5, suy ra0 ≤ r2 = 22m + ≤ 100, dẫn đến ≤ m ≤ Từ ta tìm m = nên r = r = −7 Từ 22n + 27 = e2 = 22(22t2 + 2tr) + 49 suy hai nghiệm n1 = 22t2+ 14t +

n2 = 22t2− 14t + 1, t số tự nhiên tùy ý đpcm

Chú ý tồn cấp số cộng a + bn không chứa số phương + 22n

Bài toán 1.8 (xem [1],[3]) Tam thức bậc hai chứa số phương

a) Tìm số ngun n cho n2+ 4n + 25 số phương Từ cách tìm số phương n2+ 2kn + c

b) Tìm số nguyên n cho n2+ 3n + 11 số phương Từ cách tìm số phương n2+ (2k + 1)n + c

Lời giải

a) Giả sử n2+ 4n + 25 = e2 có (n + 2)2+ 21 = e2 nêne2− (n + 2)2 = 21, (e − n − 2)(e + n + 2) = 1.3.7 Xét trường hợp sau

1)  

e + n + = 21 e − n − =

⇔  

 e = 11 n =

Lúc có 82+ 4.8 + 25 = 121 = 112

2)  

e + n + = e − n − =

⇔  

 e = n =

Lúc có 02+ 3.0 + 25 = 52

Giả sử n2+ 2kn + c = e2 có (n + k)2+ c − k2 = e2 nên e2− (n + k)2 = c − k2, (e − n − k)(e + n + k) = c − k2 Vì e − (n + k) e + (n + k) có tính chẵn lẻ nên cần phân tích số c − k2 thành tích hai số tính chẵn lẻ

(28)

Nếu k2 − c = u.v ≥ với ≤ u ≤ v u + v chẵn giải hệ phương trình n + k − e = u e + n + k = v tìm nghiệm (e, n)

b) Giả sử n2+ 3n + 11 = e2 4n2+ 12n + 44 = 4e2 có (2n + 3)2+ 35 = 4e2 nên 4e2− (2n + 3)2 = 35, hay là (2e − 2n − 3)(2e + 2n + 3) = 1.5.7 Xét trường

hợp sau

1)  

2e + 2n + = 35 2e − 2n − =

⇔  

 e = n =

Lúc có 72+ 3.7 + 11 = 81 = 92

2)  

2e + 2n + = 2e − 2n − =

⇔  

 e = n = −1

Lúc có 12− 3.1 + 11 = = 32.

Giả sử n2+ (2k + 1)n + c = e2 4n2+ 4(2k + 1)n + 4c = 4e2 có

(2n + 2k + 1)2+ 4c − (2k + 1)2= 4e2 nên 4e2− (2n + 2k + 1)2= 4c − (2k + 1)2, hay

là (2e − 2n − 2k − 1)(2e + 2n + 2k + 1) = 4c − (2k + 1)2

Vì 2e − (2n + 2k + 1) e + (2n + 2k + 1) có tính chẵn lẻ nên cần phân tích số 4c − (2k + 1)2 thành tích hai số tính chẵn lẻ

Nếu 4c − (2k + 1)2 = u.v ≥ với ≤ u ≤ v u + v chẵn giải hệ phương trình 2e − 2n − 2k − = u 2e + 2n + 2k + = v tìm nghiệm (e, n)

Nếu (2k + 1)2− 4c = u.v ≥ với ≤ u ≤ v u + v chẵn giải hệ phương trình 2n + 2k + − 2e = u 2e + 2n + 2k + = v tìm nghiệm (e, n).đpcm Bài tốn 1.9 Chứng minh số sau khơng số phương:

a) Tích hai số nguyên dương chẵn liên tiếp

b) Tích bốn số nguyên dương liên tiếp

Lời giải

a) Giả sử 2a(2a + 2) = b2 số b phải chẵn, tức b = 2c Thay vào đẳng thức a(a + 1) = b2 Vì (a, a + 1) = nên theo tính chất 1.3 phải có a = c2, a + = e2, c ≥ Dễ thấy c2 < c2+ < c2+ 2c + = (c + 1)2 nên theo tính chất 1.7 khơng tồn số phươnga + = c2+ = e2, trái với điều giả sử

(29)

a) Tích hai số nguyên dương liên tiếp

b) Tích hai số nguyên dương lẻ liên tiếp

c) Tích ba số nguyên dương liên tiếp

Lời giải

a) Giả sử a(a + 1) = bn Vì (a, a + 1) = nên theo tính chất 1.3 phải có a = cn, a + = en, c ≥ 1.Với n ≥ ta rằngcn < cn+ < (c + 1)n, tức có

a = cn < a + = en = cn+ < (c + 1)n, theo tính chất 1.7 khơng tồn số a + = en

Ta chứng minh quy nạp theo n cn < cn + < (c + 1)n Với n = c2 < c2+ < c2+ 2c + = (c + 1)2, khẳng định

Giả sử khẳng định đến n, xét số mũ n + có cn + + < c.cn + c =< (cn+ 1)c < (c + 1)n(c + 1) < (c + 1)n+1 , khẳng định với n +

Vậy khẳng định với số nguyên dươngnbất kì nên không tồn sốa+1 = en b) Xét số lẻ a giả sử a(a + 2) = bn Đặt d = (a, a + 2) d ước (a + 2) − a = 2, a lẻ nên d = Theo tính chất 1.3 phải có a = cn, a + = en, c ≥ Với n ≥ 2, ta cn < cn+ < (c + 1)n, tức cóa = cn < a + = en = cn+ < (c + 1)n, theo tính chất 1.7 khơng tồn số a + = en

Ta chứng minh quy nạp theo n cn < cn + < (c + 1)n Với n = c2 < c2+ < c2+ 2c + = (c + 1)2, khẳng định

Giả sử khẳng định đến n, xét số mũ n + 1có cn+1+ < c.cn+ cn+ 2c + < (cn+ 2)(c + 1) < (c + 1)n(c + 1) < (c + 1)n+1 , khẳng định vớin + Vậy khẳng định với số nguyên dương n nên khơng tồn số a + = en

c) Giả sử a(a + 1)(a + 2) = bn Do (a + 1, a(a + 2)) = nên theo tính chất 1.3 phải cóa + = cn, a(a + 2) = en, đón ≥ vàc ≥ Từ đó1 = (a + 1)2− a(a + 2) = c2n − en = (c2− e)(c2n−2+ c2n−4e + · · · + c2en−2+ en−1), vế phải đẳng

thức lớn n ≥ 2,đpcm

Ghi 1) Các nhà toán hc P Erdăos v J.L.Selfridge ó chng minh c

rằng: Tích n(n > 1) số nguyên dương liên tiếp không số lũy thừa

(30)

dương lẻ liên tiếp không số lũy thừa

3) Xét tích hai số nguyên dương chẵn liên tiếp2a(2a + 2) = b3 ⇔ (2a + 1)2 = b3+ 1.

Ta biết có đẳng thức 32= 23+

Năm 1844 nhà toán học người Bỉ C E.Catalan nêu giả thuyết: Hai số

nguyên dương liên tiếp khác khơng thể số lũy thừa Nhiều

nhà tốn học tìm cách chứng minh giả thuyết này, đến năm 2002 điều

mới tiến sĩ Preda Mihailescu chứng minh đầy đủ (http://www.math.uni

Paderborn de/ preda/ papers/ caterelle.ps)

Bài toán 1.11 Chứng minh số sau khơng số phương:

a) Tổng bình phương hai số lẻ

b) Tổng lũy thừa bậc chẵn hai số lẻ

Lời giải

a) b) Có dạng 4k +

Bài tốn 1.12 (xem [1],[3]) Chứng minh tổng bình phương k số nguyên dương liên tiếp không số phương với số k 3, 4, 5, 6, 7, 8, 9, 10

Lời giải

a) Với n ≥ 1, xét tổng Sk = n2+ (n + 1)2+ · · · + (n + k − 1)2 = 12+ 22+ · · · +

(n + k − 1)2− (12+ 22+ · · · + (n − 1)2) = 12+ 22+ · · · + (k − 1)2+ k2+ (k + 1)2−

12+ (k + 2)2− 22+ · · · + (k + n − 1)2− (n − 1)2.

Chú ý 12+ 22+ · · · + (k − 1)2 =

6(k − 1)k(2k − 1)

Còn k2+ k(k + 2) + k(k + 4) + · · · + k(k + 2n − 2) = nk2+ 2k(1 + + · · · + (n − 1)) = nk2+ kn(n − 1) = kn2+ k(k − 1)n Từ Sk = kn2+ k(k − 1)n +

1

6(k − 1)k(2k − 1) Từ áp dụng tính chất 1.2, định lý 1.17 cho trường hợp sau

* S3= 3n2+ 6n + = 3(n + 1)2+ 2, có dạng 3m +

* S4 = 4n2+ 12n + 14 = 4(n2+ 3n + 3) + 2, có dạng 4m +

* S5 = 5n2+ 20n + 30 = 5((n + 2)2+ 2) Số có dạng m2+ khơng chia hết cho

khi đặt m = 5t + r với r = 0, 1, 2, −1, −2

* Nếu S6 = 6n2 + 30n + 55 = 6(n + 2)(n + 3) + 18 + = m2 số m lẻ nên có

(31)

7 đặt n = 7t + r với r = 0, 1, 2, 3, −1, −2, −3

*S8 = 4(2n2+ 14n + 35) Nếu 2n2+ 14n + 35 = m2 số m lẻ nên có 2(n + 3)(n +

4) + 10 = (m − 1)(m + 1) Vế phải chia hết cho 4, 10 = 2.5 * S9 = 9(n + 4)2+ 9.6 + 6, có dạng 9m +

*S10 = 5(2n2+ 18n + 57) Số 2n2+ 18n + 57 = 2(n2− n + 1) + 20n + 55 không chia

hết cho đặt n = 5t + r với r = 0, 1, 2, −1, −2

Ghi 1) Với k = phương trình n2+ (n + 1)2 = m2⇔ (2n + 1)2− 2m2= −1 ⇔ t2− 2m2 = −1 ( Phương trình Pell ) với t = 2n + 1, có vơ hạn nghiệm nguyên dương, chẳng hạn 32+ 42= 52 202+ 212 = 292 Dễ dàng chứng minh rằng:

Nếu phương trình n2 + (n + 1)2 = m2 có nghiệm (n1; n1+ 1; m1) có

nghiệm (n2; n2+ 1; m2) với n2 = 3n1+ 2m1+ 1; m2 = 4n1+ 3m1+

Mệnh đề đảo đúng, tức hệ thức với 32+ 42 = 52 nghiệm phương trình n2+ (n + 1)2 = m2

2) Với k = 11 mệnh đề khơng Chẳng hạn:

182+ 192+ 202+ 212+ 222+ 232+ 242+ 252+ 262+ 272+ 282= 772 Bài toán 1.13 (xem [1],[3]) Chứng minh :

a) Tổng lũy thừa bậc chẵn ba số nguyên liên tiếp không số lũy thừa

bậc chẵn

b) Tổng lũy thừa bậc chẵn số nguyên liên tiếp không số

lũy thừa

Lời giải

a) Ba số nguyên liên tiếp chia cho có số dư khác nên tổng

lũy thừa bậc chẵn chúng có dạng 3n + Áp dụng định lý 1.17

b) Viết số dạng 9t + r với −4 ≤ r ≤ tổng lũy thừa bậc chẵn 2n số nguyên liên tiếp có dạng S = 9m + 2(1n + 4n + 16n) = 9k + 2(1n+ 4n+ 7n)

Đặt n = 3v + s với ≤ s ≤ 1n+ 4n+ 7n có dạng 9u + nên S có dạng 9x + Áp dụng tính chất 1.2.đpcm

(32)

Lời giải Tổng bình phương n số tự nhiên liên tiếp từ đến n S = 12+ 22+ · · · + n2 =

6n(n + 1)(2n + 1)

Giả sử S = m2 n(n + 1)(2n + 1) = 6m2 (1.4) Xét trường hợp sau

1) Nếu n = 6k (1.4) có dạng k(6k + 1)(12k + 1) = m2 Các thừa số vế trái nguyên tố sánh đôi nên áp dụng liên tiếp định lý tích k(6k + 1) tích k(6k + 1)(12k + 1) thừa số số phương Với k = vế trái 7.13 nên khơng số phương Với k = vế trái có dạng 4k + nên khơng số phương Với k = vế trái có dạng 9k + nên khơng số phương Với k = n = 24, vế trái 4.25.49 nên có 12+ 22+ · · · + 242 = 702 (1.5)

2) Nếu n = 6k + (1.4) có dạng(6k + 1)(3k + 1)(2k + 1) = m2 Lập luận tương tự trên, ý số 2k + nhỏ (lớn 1) số phương khik = 4, lúc n = 25 > 24

3) Nếu n = 6k + (1.4) có dạng (3k + 1)(2k + 1)(12k + 5) = m2 4) Nếu n = 6k + (1.4) có dạng (2k + 1)(3k + 2)(12k + 7) = m2 5) Nếu n = 6k + (1.4) có dạng (3k + 2)(6k + 5)(4k + 3) = m2 6) Nếu n = 6k + (1.4) có dạng (6k + 5)(k + 1)(12k + 1) = m2 Cả bốn trường hợp không xảy đẳng thức k =

Xét k > 0, lập luận thừa số vế trái trường hợp số phương (*)

Chú ý 12k + = 3(4k + 1) + 2và 6k + = 3(2k + 1) + 2nên tích vế trái trường hợp chứa thừa số 3t + 2, thừa số khơng số phương theo định lý 3.3, trái với (*)

Như từ đẳng thức (1.5) lập luận rút sốn = 24là số nhỏ thỏa mãn đề đpcm

(33)

Chương Bất đẳng thức số học

Trong chương ta xét số bất đẳng thức tính tốn liên quan đến

tập rời rạc bất đẳng thức lớp hàm số học

2.1 Bất đẳng thức tập số nguyên

Bài toán 2.1 Cho n số nguyên dương Chứng minh p

C1 n+

p C2

n+ · · · +

p Cn

n ≤

p

n(2n − 1).

Lời giải Áp dụng bất đẳng thức Cauchy, ta có

p C1

n+

p C2

n+ · · · +

p Cn

n

2

≤ (Cn1+ Cn2+ · · · + Cnn)(1 + + · · · + 1) Ta có

Cn1+ Cn2+ · · · + Cnn = 2n− nên

p C1

n+

p C2

n+ · · · +

p Cn

n ≤

p

n(2n − 1).

Dấu đẳng thức xảy Cn1= Cn2 = · · · = Cnn ⇔ n =

Bài toán 2.2 Chứng minh với ≤ k ≤ n k, n ∈ Z ln có bất đẳng thức:

C2n+kn C2n−kn ≤ (C2nn )2 Lời giải Cố định n, ta xét dãy số:

uk = C2n+kn C2n−kn với ≤ k ∈ Z

(34)

Ta chứng minh dãy uk đơn điệu giảm Thật vậy, ta có

uk+1 < uk

⇔ (2n + k + 1)! n!(n + k + 1)!

(2n − k − 1)! n!(n − k − 1)! < (2n + k)!

n!(n + k)!

(2n − k)! n!(n − k)! ⇔ 2n + k +

n + k + <

2n − k n − k ⇔ n + 2nk >

Suy uk ≤ u0 với ≤ k ∈Z⇔ C2n+kn C2n−kn ≤ (C2nn )2

Bài toán 2.3 Với n số nguyên dương, chứng minh

n(C

1

n+ 2Cn2+ 3Cn3+ · · · + nCnn) ≤ n!

Lời giải Với x, với n số nguyên dương, ta có

(1 + x)n = Cn0+ Cn1x + Cn2x2+ · · · + Cnn−1xn−1+ Cnnxn Lấy đạo hàm theo x hai vế biểu thức ta

n(1 + x)n−1 = Cn1+ 2Cn2x + · · · + (n − 1)Cnn−1xn−2+ nCnnxn−1 Thay x = 1, vào ta

n.2n−1 = Cn1+ 2Cn2+ 3Cn3+ · · · + nCnn Khi bất đẳng thức chuyển dạng

2n−1≤ n!

Ta chứng minh bất đẳng thức phương pháp quy nạp

Thật vậy, bất đẳng thức vớin = 1.Giả sử bất đẳng thức vớin = k, k nguyên dương tức là: 2k−1 ≤ k! Ta phải chứng minh bất đẳng thức với n = k + 1, tức phải chứng minh 2k+1 ≤ (k + 1)!

(35)

Bài toán 2.4 Cho n số nguyên dương lớn Chứng minh < 1 +

n n

<

Lời giải Áp dụng công thức khai triển Newtơn, ta có

 +

n n

= Cn0+ Cn1.1 n + C

2 n

1 n

2

+ · · · + Cnk.1 n

k

+ ˙+Cnn1 n

n Ta có Cn0 = 1, Cn1.1

n = Do 

1 + n

n > Ta lại có

Cnk nk =

n!

nk.k!.(n − k)! ≤

1 k! ≤

1 (k − 1)k =

1 k − −

1 k Áp dụng bất đẳng thức ứng với k = 2, 3, , n, ta thu

Cn2

n2 ≤ −

1 Cn3

n3 ≤

1 −

1

Cnn nn ≤

1 n − 1−

1 n Suy

 +

n n

= Cn0+ Cn1.1 n +

n

X

k=2

Cnk

nk < + −

1 n < 3, điều phải chứng minh

Bài toán 2.5 Chứng minh C2017k + C2017k+1 ≤ C20171008 + C20171009 với ≤ k ≤ 1008, k ∈N

Lời giải Nhận xét rằng, bất đẳng thức cần chứng minh có dạng:C2018k+1 ≤ C1009 2018

Xét dãy số uk = C2018k+1 với ≤ k ≤ 1008, k ∈N

Ta chứng minh dãy uk dãy đơn điệu tăng Thậy vậy, xét uk+1 > uk

C2018k+2 > C2018k+1

⇔ 2018!

(k + 2)!(2016 − k)! >

2018!

(k + 1)!(2017 − k)! ⇔

(36)

⇔ 2015 > 2k ⇔ k ≤ 1007 ⇒ uk ≤ u1008∀k ≤ 1008

⇔ C2018k+1 ≤ C20181009, ≤ k ≤ 1008, k ∈N Bài toán 2.6 Cho ≤ n ∈N Chứng minh

Cn0.Cn1 .Cnn ≤2

n− 2

n − n−1

Lời giải Ta có Cn0.Cn1 .Cnn = Cn1 .Cnn−1

Áp dụng bất đẳng thức AM-GM, ta có

Cn1 .Cnn−1 ≤C

1

n+ Cn2+ · · · + Cnn−1

n −

n−1 =

2n− n −

n−1 Bài toán 2.7 Với n số nguyên dương, n ≥ Chứng minh

1 n(C

1

n+ 2Cn2+ 3Cn3+ · · · + nCnn) < n!

Lời giải Với x, với n số nguyên dương, ta có (1 + x)n = Cn0+ Cn1x + Cn2x2+ · · · + Cn.n Lấy đạo hàm hai vế đẳng thức ta

n(1 + x)n−1 = Cn1+ 2Cn2x + · · · + (n − 1)Cnn−1xn−2+ nCnnxn−1 Thay x = vào hệ thức trên, ta

n.2n−1 = Cn1+ 2Cn2+ 3Cn3+ · · · + nCnn Khi bất đẳng thức cần chứng minh chuyển dạng:

2n−1 < n!(∗)

Ta chứng minh bất đẳng thức phương pháp qui nạp toán học

Hiển nhiên vớin = 2bất đẳng thức(∗)đúng Giả sử(∗)đúng vớin = k, k ≥ 2, k số nguyên dương, tức là: 2k−1 < k! Ta chứng minh (∗) với n = k + 1,tức phải chứng minh 2n < (n + 1)!

(37)

2.2 Bất đẳng thức lớp hàm số học

Bài toán 2.8 Cho n số tự nhiên lớn Kí hiệu σ(n) tổng tất ước tự nhiên n ( kể n), cịn kí hiệu ϕ(n) số lượng số nhỏ n nguyên tố với n Chứng minh với n ≥ 2, ta có

σ(n) + ϕ(n) ≥ 2n

Lời giải Giả sử = d1 < d2 < d3 < < dk = n ước tự nhiên số

n Trong số tự nhiên không vượt n, số lượng bội di

n di

; số

lượng số không lớn n không nguyên tố với n theo định nghĩa n − ϕ(n) Lại theo định nghĩa σ(n)

σ(n) = d1+ d2+ + dk−1+ dk

Vì dk = n nên Ta có

d1+ d2+ + dk+1 = σ(n) − n (2.1)

Rõ ràng, ta có n d1

= dk; n d2

= dk−1; ; n dk

= d1 Suy

n d2

+ n d3

+ + n

dk = dk−1+ dk−2+ + d1, (2.2) n

d2

+ n d3

+ + n dk

là số lượng tất bội số ước số d2, d3, , dk

cộng lại (chú ý a vừa bội số ước di dj với i 6= j

a tổng có mặt hai lần)

Do số lượng ước không lớn n không nguyên tố với n không vượt số n

d2

+ n d3

+ + n dk

, tức

n d2

+ n d3

+ + n dk

≥ n − ϕ(n) (2.3)

Từ (2.1), (2.2) (2.3), suy

σ(n) − n ≥ n − ϕ(n), hay σ(n) + ϕ(n) ≥ 2n

(38)

nguyên tố với p 1, 2, , p − 1) Điều có nghĩa p ngun tố

σ(p) + ϕ(p) = 2p Tóm lại với n ≥ 1, ta có

σ(n) + ϕ(n) ≥ 2n (2.4)

Dấu ” = ” (2.4) xảy n số nguyên tố

Bài toán 2.9 Chứng minh bất đẳng thức σ(n) > 3n với tập hợp vô hạn số tự nhiên n

Lời giải Rõ ràng d ước số n, n

d ước số n Vì σ(n) = d1+ d2+ + dk = n

 d1

+ d2

+ + dk



Trong d1, d2, , dk tất ước n Lấy n số tùy ý cho

là bội số 16! = 1.2.3 .16 Số số n dĩ nhiên vơ hạn Nói riêng ước n có 1, 2, 3, , 16,

σ(n) = n 

1 d1

+ d2

+ + dk



≥ n1 + +

1

3 + + 16



(2.5) Do

1 + +

1

3 + +

16 = + + 1 +  + 1

5 + +

 +

1

9+ + 16



nên hiển nhiên ta có

1 3+

1 >

1 = 2; 5+ 6+ 7+ >

1 = 2; 9+

10+ + 16 >

1 16 =

1 nên

1 + +

1

3 + +

16 > (2.6)

Từ (2.5) (2.6) suy σ(n) > 3n

(39)

Bài toán 2.10 Chứng minh tồn vô hạn số tự nhiên n cho bất đẳng thức

σ(n) n >

σ(k) k với k = 1, 2, , n −

Lời giải Giả thiết phản chứng tồn hữu hạn giá trị n ∈N thỏa mãn bất đẳng thức

σ(n) n >

σ(k)

k , ∀k = 1, 2, , n − Giả sử N số lớn giá trị n

Đặt

An = max

σ(i)

i , n = 1, 2, ; ≤ i ≤ n Rõ ràng ta có A1≤ A2 ≤ ≤ AN

Xét n > N Theo định nghĩa số N ∃k0 ∈ {1, 2, , n − 1} cho

σ(n) n >

σ(k0)

k0

Từ kết hợp với m > n Am ≥ An suy

AN = AN +1= AN +2 =

Như dãy số {An}, n = 1, 2, bị chặn số AN =

σ(N )

N (theo định nghĩa số N ∀i = 1, 2, , N − ta có σ(N )

N > σ(i)

i )

Xét số 2N, ta thấy ước số 2N có dạng 2d (với N d) số Như

σ(2N ) ≥ 2σ(N ) + ⇒ σ(2N ) 2N ≥

2σ(N ) + 2N =

σ(N ) N +

1 2N >

σ(N ) N Bất đẳng thức σ(2N )

2N > σ(N )

N mâu thuẫn với việc dãy {An} bị chặn σ(N )

N Điều vô lý chứng tỏ giả thiết phản chứng sai, suy tồn vô hạn số tự nhiên n ∈ N cho bất đẳng thức σ(n)

n > σ(k)

(40)

Bài tốn 2.11 Kí hiệud(n) trung bình cộng tất ước số củan (kể n) Chứng minh với n, Ta có

n ≤ d(n) ≤ n +

Lời giải Giả sử = d1 < d2 < d3 < < dk = n ước tự nhiên số n

Như vậy, ≤ di ≤ n, ∀i = 1, k

Theo định nghĩa, ta có

d(n) = d1+ d2+ + dk

k

Chú ý n d1

> n d2

> > n dk−1

> n dk

, n di

, i = 1, k tất ước n Như ta có d1 =

n dk

; d2 =

n dk−1

; ; dk−1 = n d2

; dk = n d1

Vì lẽ ta

d1dk = d2dk−1 = = didk−i+1= n, ∀i = 1, k (2.7)

Bây ta chứng minh vế trái bất đẳng thức Ta có

d(n) = d1+ d2+ + dk

k =

2(d1+ d2+ + dk)

2k =

d1+ dk

2 +

d2+ dk−1

2 + +

dk−1+ d2

2 +

dk+ d1

2

k

Từ theo bất đẳng thức Cauchy, suy

d(n) ≥ √

d1dk +

p

d2dk−1+ +

p

dk−1d2+

√ dkd1

k (2.8)

Từ (2.7) (2.8), suy

d(n) ≥ k √

n n =

√ n

Vậy d(n) ≥ √n Vế trái bất đẳng thức chứng minh Bây ta xét vế phải Do di ≥ 1, ∀i = 1, k, nên ta có

0 ≤ (di− 1)(dk−i+1− 1) = didk−i+1+ − di− dk−i+1 (2.9)

Từ (2.7) (2.9) suy với i = 1, 2, , k ta có n + − (di+ dk−i+1) ≥ 0hay

(41)

Cộng vế k bất đẳng thức dạng (2.10), ta được:

2

k

X

i=1

di≤ k(n + 1) ⇔

1 k

2

X

i=1

di≤

n +

Theo định nghĩa d(n) điều có nghĩa d(n) ≤ n + Tóm lại, ta có √n ≤ d(n) ≤ n +

2

Bài toán 2.12 Cho n số tự nhiên lớn Kí hiệu σ2(n) = d21+

d22+ + d2m, với d1, d2, , dm tất ước n Chứng minh rằng:

σ2(n) < n2

√ n

Lời giải Giả sửd1 < d2 < < dmlà tất ước củanthì

n d1

> n d2

> > n dm

cũng tất ước n Vì d21+ d22+ + d2m = n2

 d21 +

1

d22 + + d2 m



(2.11)

Mặt khác, ta có

n2 

1 d21 +

1

d22 + + d2 m



≤ n2 12 +

1

22 + +

1 m2



≤ n21 12 +

1

22 + +

1 n2

 (2.12)

Tóm lại, n > nên

12 +

1

22 + +

1

n2 < +

1 1.2 +

1

2.3 + +

n.(n − 1) = = +

 −1

2  + 1 2−  + +  1

n − − n  Suy 12 +

1

22 + +

1

n2 < −

1

n (2.13)

Vìn > n số nguyên nênn ≥ suy − n <

n Vì từ (2.13) suy

12 +

1

22 + +

1 n2 <

n (2.14)

Vậy từ (2.11),(2.12) (2.14) ta có

(42)

Bài toán 2.13 Giả sửσ(n) số tất ước số tự nhiên củan Chứng minh σ2(n) < 4n, với n = 1, 2,

Lời giải Giả sử a ước số n, số b = n

a ước số n Do tất ước số n chia thành cặp, đặt tương ứng với ước a <√n, với b = n

a Ngồi thêm √

n nếu√n số nguyên ( tức n số phương) Số nhỏ hai số cặp gọi số thứ số lớn

gọi số thứ hai

Xét hai khả sau:

i) Nếu n khơng phải số phương Khi tất số thứ nhỏ √n Vì gọi d∗ số lớn số thứ d∗ <√n ⇒ d∗< [√n] ([α] để phần nguyên số α)

Vì số thứ thuộc tập {1, 2, , [√n]}, từ suy số số thứ ≤ [√n] <√n (do n khơng số phương)

Vìn khơng phải số phương nênσ(n)bằng hai lần số số thứ Do ta có bất đẳng thức

σ(n) < 2√n ⇒ σ2(n) < 4n

ii) Nếu n số phương Khi √n ước số n tất ước thứ ≤√n − Kí hiệu d∗ phần i) ta có

d∗≤√n − ⇒ d∗ ≤√n − 1

Lập luận ta có số số thứ ≤ [√n − 1] ≤√n −

Khin số phương, σ(n)bằng hai lần số số thứ cộng thêm Vì suy

σ(n) < 2(√n − 1) + ⇒ σ(n) < 2√n − < 2√n ⇒ σ2(n) < 4n Tóm lại ta ln chứng minh với i = 1, 2, σ2(n) < 4n Bài toán 2.14 Cho n > Chứng minh σ(n) < n√n

(43)

i) Nếu n = 2α Do n > nên α số nguyên ≥ Rõ ràng lúc σ(n) = σ(2α) = 20+ 21+ 22+ + 2α=

α+1− 1

2 − =

α+1− 1.

Vì α ≤ nên ta có

σ(n) = 2α+1− < 2α+1≤ 2α+ α

2 = 2α.2

α

2 = n√n Vậy bất đẳng thức cho n > n có dạng n = 2α

ii) Nếun khơng có dạng 2α (tức khơng phải lũy thừa 2) Ta chứng minh quy nạp trường hợp

Do n > nên số nhỏ khơng có dạng 2α số Lúc σ(3) = + = < 3√3

Vậy bất đẳng thức n =

Giả thiết σ(k) < k√k với k, ≤ k < n k khơng có dạng 2α Ta chứng minh

σ(n) < n√n (2.15)

Vìnkhơng chia hết cho2nênn có dạngn = mp, đómlà ngun dương, cịn p số ngun tố lẻ Dễ thấy

1 + p < p√p (2.16) Thật vậy, p = 3, + < 3√3, cịn p ≥ ta có

1 + p

p = +

p ≤ + < Mặt khác √p ≥√5 > ⇒ + p

p < √

p hay + p < p√p Vậy (2.16)

Chỉ có khả sau xảy ra:

i) Nếu m = suy n = p, mà p số nguyên tố nên σ(n) = σ(p) = + p Từ (2.15) suy trường hợp ta có σ(n) = + p < p√p = n√n

Vậy (2.15) trường hợp

(44)

Vì p số nguyên tố lẻ nên p ≥ Ta có +

p ≤ +

3 ⇒ + p ≤ Lại có 2√2.√p ≥ 2√6 > Vì

3 + p <

2√p ⇒ 3(1 + p) < 2√2.p√p = 2pp2p = n√n ⇒ σ(n) < n√n Vậy (2.15) trường hợp

iii) Nếu m ≥ Do m dạng 2α nên m ≥ m < n Theo giả thiết quy nạp ta có

σ(m) < mp(m)

Ta thấy ước số n = mp có dạngd pd với dlà ước số m Vì

σ(n) = σ(m) + pσ(m) = (p + 1)σ(m) ⇒ σ(n) < (p + 1)m√m Lại áp dụng (2.16) ta có

σ(n) < pp(p).mp(m) = mp√mp = n√n Vậy (2.15) trường hợp

Tóm lại, n khơng có dạng 2α ta ln có σ(n) < np(n) Vì bất đẳng thức σ(n) < np(n) chứng minh hồn tồn

Bài tốn 2.15 Cho A số tự nhiên Kí hiệu S(A) tổng chữ số A A viết dạng thập phân

a) Giả sử A1, A2, , An số tự nhiên Chứng minh

S(A1+ A2+ + An) ≤ S(A1) + S(A2) + + S(An)

b) Giả sử A, B cặp số tự nhiên Chứng minh S(AB) ≤ S(A).S(B) c) Cho N số tự nhiên Chứng minh S(8N )

S(N ) ≥ Lời giải

a) Ta chứng minh bất đẳng thức cho quy nạp

(45)

Giả sử A1= akak−1 a2a1, A2 = bkbk−1 b2b1 Chú ý rằng:

a1+ b1=

 

a1+ b1 a1+ b1 ≤

1α nếua1+ b1 > 10

ở α = a1+ b1− 10, vậy: a1+ b1 ≥ S(a1+ b1)

Lập luận liên tiếp dẫn đến S(A1+ A2) ≤ S(A1) + S(A2)

Giả sử bất đẳng thức cho đến n = k, tức

S(A1+ A2+ + Ak−1) ≤ S(A1) + S(A2) + + S(Ak−1) (2.17)

Xét bất đẳng thức với n = k, ta có

S(A1+ A2+ + Ak) = S [(A1+ Ak−1) + Ak] ≤ S(A1+ Ak−1) + S(Ak)

Theo giả thiết quy nạp, suy

S(A1+ A2+ + Ak) ≤ S(A1) + S(A2) + + S(Ak−1) + S(Ak)

Vậy bất đẳng thức đến n = k

Theo nguyên lí quy nạp bất đẳng thức cho với mọin Dấu” = ” xảy phép cộng A1+ A2+ + An phép cộng không nhớ

Chú ý nói riêng lấy A1= A2= = An = A, ta có với A n số

tự nhiên, ta có bất đẳng thức sau: S(nA) ≤ nS(A)

b) Giả sử B = b1b2 bk ⇒ B = bk+ 10bk−1+ 102bk−2+ + 10k−1b1

Trước hết ta có nhận xét sau: Nếu N số tự nhiên cho N = α1α2 αq,

thì

10pN = α1+ α2 αp0

| {z }

psố

, ∀p số tự nhiên

Suy S(N ) = S(10pN ) (vì α1+ α2+ + αq)

Ta có AB = Abk+ 10Abk−1+ 102Abk−2+ + 10k−1Ab1

Theo câu a)

S(AB) = S(Abk+10Abk−1+ .+10k−1Ab1) ≤ S(Abk)+S(10Abk−1)+ .+S(10k−1Ab1)

(46)

Áp dụng hệ câu a), từ (2.18) ta có

S(AB) ≤ bkS(A) + bk−1S(10A) + + b1S(10k−1A) (2.19)

Từ (2.19), ta có

S(AB) ≤ bkS(A) + bk−1S(A) + + b1S(A) = S(A)(b1+ b2+ + bk)

Vậy nên S(AB) ≤ S(A).S(B) c) Theo nhận xét trên, ta có

S(N ) = S(1000N ) = S(125.8N ) Theo câu b) suy

S(N ) ≤ S(125).S(8N ) Do S(125) = ⇒ S(N ) ≤ 8S(8N ) hay S(8N )

S(N ) ≥

Bài toán 2.16 Cho số tự nhiên n > Kí hiệu ν(n) ước số nguyên tố n

a) Chứng minh tồn vô hạn số n có dạng n = 2k cho ta có bất đẳng thức ν(n) < ν(n + 1)

b) Chứng minh tồn vô hạn sô tự nhiên n cho ta có ν(n) < ν(n + 1) < ν(n + 2)

Lời giải Giả sử có số n0 = 2k0 cho ta có bất đẳng thức ngược lại

ν(n0) ≥ ν(n0+ 1) (2.20)

Rõ ràng ν(n0) = ν(2k0) = (vì 2k0 có ước nguyên tố 2) Cịn

ν(n0+ 1) = ν(2k0+ 1) ≥ (vì 2k0+ có ước nguyên tố)

Từ (2.20) suy

1 = ν(n0) ≥ ν(n0+ 1) ≥ ⇒ ν(2k0 + 1) = (2.21)

Như từ (2.21) suy

2k0 + = pm, với m ∈

(47)

Có hai khả xảy ra:

i) Nếu m = 2l (m số chẵn), từ (2.22) ta có 2k0 = p2l− = (pl− 1)(pl+ 1),

suy pl− pl+ lũy thừa Điều xảy 

pl+ = pl− =

⇒  

p = l = k0=

ii) Nếu m = 2l + (m số lẻ) ⇒ m = Thật vậy, m > từ (2.22) ta có

2k0 = pm− = (p − 1)(pm−1+ pm−2+ + p + 1

| {z }

msố

)

suy pm−1+ pm−2+ + p + lũy thừa

Do p số nguyên tố nên p lẻ pm−1+ pm−2+ + p tổng m − số lẻ; m lẻ nên pm−1+ pm−2+ + p chẵn, suy pm−1+ pm−2+ + p + lẻ, suy vơ lí

Nếu p = pm−1+ pm−2+ + p + lẻ, suy vô lí Do khơng thể có m > 1, suy m = Vậy từ (2.22) Ta có

2k0 + = p. (2.23)

Từ (2.23) suy rak0 = 2q0 vớiq0 nguyên dương Thật vậy, trái lại thìk0 = 2q0r

với r > số lẻ nên số nguyên tố

p = 2k0 + = 22q0r+ = 0( (mod 22q0) + 1) ⇒ p .22k0 + < p.

Điều trái với giả thiết p số nguyên tố

Như ν(n0) = ν(n0+ 1) với n0 = 2k0 k0 hoặc k0 = 2q0

với q0 nguyên dương Điều có nghĩa ∀k 6= 3, k 6= lũy thừa (và

số k vô hạn) ta ln có bất đẳng thức ν(n) < ν(n + 1)

(48)

Xét hai tập hợp:

K1= {k ∈ N: ν(2k) < ν(2k+ 1)}; K2= {k ∈ N: ν(2k+ 1) < ν(2k+ 2)}

Ta có tập K1∩ K2 có hữu hạn phần tử Do tồn số q0 đủ lớn cho

ứng với sốk0 = 2q0 (dĩ nhiên > 5) xét số k = k0+ 1, k0+ 2, 2k0− =

2q0+1 − < 2q0+1 ( số lũy thừa 2 6= 3, nên

chúng thuộc K1) Chú ý đến q0 chọn đủ lớn nên số thuộcK2 (vì

K1∩ K2 có hữu hạn phần tử), tức ta có

ν(2k+ 1) ≥ ν(2k+ 2)∀k = k0+ 1, k0+ 2, , 2k0−

Chú ý

ν(2k + 2) = ν2 2k−1+ 1= + ν(2k−1+ 1) nên

ν(2k+ 1) ≥ + ν(2k−1+ 1)∀k = k0+ 1, k0+ 2, , 2k0−

Suy

ν(22k0−1+ 1) ≥ + ν(22k0−2+ 1) ≥ ≥ (k

0− 1) + ν(2k0+ 1) ≥ k0

(do ν(2k0+ 1) ≥ 1)

Kí hiệu p1 < p2 < p3 dãy tất số nguyên tố Bất đẳng thức

ν(22k0−1+ 1) ≥ k

0 có nghĩa số ước nguyên tố 22k0−1+ khơng

k0 Vì theo nguyên tắc biểu diễn số thừa số nguyên tố Ta có

22k0−1+ ≥ p

1p2 pk0 = (2.3.4.7.11)(p6 pk0) >

5.4k0−5.

Suy

22k0−1+ > 22k0 > 22k0−1+ 1.

Nhưng 22k0−1+ > 22k0−1+ 1 là vô lý Suy giả thiết phản chứng sai Vậy số

(49)

Bài toán 2.17 Cho số tự nhiên k ≥ Kí hiệu g(k) ước số lẻ lớn k Chứng minh với số tự nhiên n ≥ 1, ta có

0 < n X k=1 g(k) k − 2n < Lời giải Theo định nghĩa g(k) ta có

k = 2m(k).g(k) ⇒ g(k) k =

1 2m(k)

Vậy S = n X k=1 g(k) k = n X k=1

2m(k) (2.24)

Chú ý với số tự nhiên n, 1, 2, , n có n

2 số chẵn n 22 số

là bội 4, n

23 số bội 8, tổng quát có

n

2m số bội m.

Rõ ràng với n ln tồn số tự nhiên M cho h n

2M

i

=h n

2M +1 = =

i

(2.25)

Từ lập luận suy số k(1, 2, , n) để m(k) nhận giá trị m h n

2m

i

−h n 2m+1

i

(2.26)

Từ (2.24),(2.25) (2.26) ta có

S = M X m=0 2m h n 2m i

−h n 2m+1 i = M X m=0 2m h n 2m i − M X m=0 2m+1 h n 2m i

( h n 2M +1

i = 0) Suy

S =hn 20 i + M X m=1  2m −

1 2m−1

 h n 2m

i

= n −

M X m=1 2m h n 2m i (2.27) Rõ ràng với α ta ln có [α] ≤ α, từ (2.27) suy

S ≥ n −

M X m=1 2m n

2m = n − n M

X

m=1

1 4m

Chú ý rằng:

M

X

m=1

1 4m =

1 − 4M

3 ⇒ S ≥ n − n

 −

4M

 =

3n + n 3.4M >

(50)

⇒ S −2 3n > ⇒ <

n X k=1 g(k) k − 2n

tức phần bên trái bất đẳng thức kép chứng minh

Bây ta chứng minh phần bên phải bất đẳng thức kép cho Trước

hết ta có nhận xét sau đây: ∀p, q ∈N, thì 

p q 

= p +

q − (2.28)

Thật vậy, đặt 

p q 

= r, p, q ∈ N nên p = rq + s với s = 0, 1, , q − Suy

 p q 

= r = p − s q ≥

p − (q − 1)

q =

p + q − Vậy (2.28)

Áp dụng (2.28) suy

S = n −

M X m=1 2m h n 2m i n − M X m=1 2m

n + 1 2m −

 (2.29) Ta có n − M X m=1 2m

n + 2m −



= n − (n + 1)

M

X

m=1

1 4m +

M

X

m=1

1

2m (2.30)

= n − n +

 −

4M 

+ 

1 + 2M



= 3n +

2 3+

n + 3.4M −

1 2M

Thay (2.30) vào (2.29) ta

S ≤ 3n +

2 +

n + 3.4M −

1

2M (2.31)

Theo cách xác định số M, ta có 2M > n Rõ ràng n > n +

3 (do n ∈ N) suy n +

3.4M <

2M Vậy từ (2.31) ta có S < 3n +

2

Bài toán 2.18 Nếu n k số nguyên dương đặt Tn(k) = kk

k k

( lũy

(51)

Lời giải Trước hết ta chứng minh bổ đề sau

Bổ đề: Ta có

Tn+1(3) > Tn+2(2) > 4Tn(3) (2.32)

Chứng minh bổ đề: Ta chứng minh quy nạp

- Với n = 1,

Tn+1(3) > Tn+2(2) > 4Tn(3) ⇔ T2(3) > T3(2) > 4T1(3) (2.33)

⇔ 33 > 222 > 4.3 ⇔ 27 > 16 > 12 Vì (2.33) đúng, (*) n =

- Giả sử bất đẳng thức (*) đến n = k, tức có

Tk+1(3) > Tk+2(2) > 4Tk(3) (2.34)

-Xét n = k + Ta có

T(k+1)+1= Tk+2(3) = 3Tk+1(3). (2.35)

Theo (2.34) suy

Tk+2(3) > 3Tk+2(2)> 2Tk+2(2) = Tk+3(2)

Lại có Tk+3(2) = 2Tk+2(2) theo giả thiết quy nạp (2.34) suy

Tk+3(2) > 24Tk(3) = 22Tk(3).4Tk(3)

Rõ ràng Tk(3) ≥ 3, ∀k, 4Tk(3) = Tk+1(3), từ suy

Tk+3(2) > 26Tk+1(3) > 4Tk+1(3) (2.36) Từ (2.35), (2.36) suy

Tk+2(3) > Tk+3(2) > 4Tk+1(3)

Vậy (*) n = k +

(52)

Từ bổ đề nói riêng suy

Tn+1(3) > Tn+2(2) > Tn(3) (2.37)

Áp dụng (2.37) với n ≤ 1995, ta có

T1997(2) > T1995(3) > T1994(3) > > T1(3)

Dãy bất đẳng thức chứng tỏ từ Tn(3) > T1997(2) suy n ≤ 1996

Chú ý: Dĩ nhiên lại dựa vào (2.37) ta có

T1997(2) < T1996(3) < T1997(3) < T1997(3) <

Vì ta khẳng định

Tn(3) > T1997(2) ⇔ n ≤ 1996

Bài toán 2.19 Hàm số f (x, y) xác định với cặp số nguyên không âm (x, y) thỏa mãn với cặp số nguyên không âm (x, y):

a) f (0, y) = y + 1; b) f (x + 1, 0) = f (x, 1);

c) f (x + 1, y + 1) = f (x, f (x + 1, y)) Cho k số nguyên không âm cho

[f (4, 2000) − f (3, 2000)] 2k Chứng minh k ≤ 2003

Lời giải Từ a), b), c) trực tiếp suy

f (1, 0) = 2; f (2, 0) = 3; f (3, 0) = 5; f (4, 0) = 13 (2.38) Ta chứng minh với n = 1, 2,

(53)

f (2, n) = 2n + 3; (2.40)

f (3, n) = 2nn+3− (2.41) Thật vậy, (2.39),(2.40),(2.41) n = (theo (*))

Giả sử (2.39),(2.40),(2.41) n = k tức

f (1, k) = k + 2; f (2, k) = 2k + 3; f (3, k) = 2kk+3− (2.42) Xét n = k + Theo tính chất a), c) giả thiết quy nạp (2.42) Ta có

f (1, k + 1) = f (0, f (1, k)) = f (1, k) + = (k + 2) + = k + = (k + 1) + Vậy (2.39) n = k + 1, theo nguyên lý quy nạp (2.39) ∀n ∈N

Ta có f (2, k + 1) = f (1, f (2, k)) Vì (2.39) ∀n ∈N, nên có f (2, k + 1) = f (2, k) +

Theo giả thiết quy nạp có

f (2, k + 1) = 2k + + = 2(k + 1) +

Vậy (2.40) n = k + Theo nguyên lí quy nạp (2.40) ∀n ∈ N. Ta lại có f (3, k + 1) = f (2, f (3, k)) Vì (2.40) ∀n ∈ N nên ta có

f (3, k + 1) = 2f (3, k) + Theo gải thiết quy nạp

f (3, k + 1) = 2(2k+3− 3) + = 2k+4−

Vậy (2.41) n = k + Theo nguyên lí quy nạp (2.41) Do (2.39),(2.40),(2.41) chứng minh

Cũng quy nạp tương tự, ta chứng minh với y ngun khơng âm

(54)

Theo tính chất c)

f (4, 2000) = f (3, f (4, 1999)) Theo (2.41)

f (3, f (4, 1999)) = 2f (4,1999)+3− Ta lại có

f (4, 2000) = 2f (4,1999)+3− (2.44) Áp dụng liên tiếp (2.43) ta có

f (4, 1999) > f (4, 1998) > f (4, 1997) > > f (4, 0) = 13 (2.45) Vì f (4, j) số nguyên dương (j = 0, 1999), nên từ (2.45) suy

f (4, 1999) > 2000 (2.46) Theo (2.44), (2.41) ta

f (4, 2000) − f (3, 2000) = (2f (4,1999)+3− 3) − (22003− 3) (2.47) = −22003+ 2f (4,1999)+3

= 22003 h

2f (4,1999)−2000− 1i

Từ (2.46) suy 2f (4,1999)−2000 − số nguyên dương lẻ.Vì f (4, 2000) − f (3, 2000) chia hết cho 22003 không chia hết cho 2l với l > 2003 Do từ f (4, 2000) − f (3, 2000) chia hết cho 2k suy k ≤ 2003

Bài toán 2.20 Với số nguyên dương n, kí hiệu σ(n) tổng tất ước tự nhiên n ( kể n ) Với số thực dươngx, kí hiệu

Nx= {n : n nguyên dương , n ≤ x, σ(n) ≥ 2n}

Đặt Tx = |Nx|, qua |A| kí hiệu số phần tử tập hợp A Chứng minh

rằng với số nguyên dương, ta có bất đẳng thức sau:

(55)

-Nếu [x] ≤ 1, [x] phần nguyên số x Với n ∈ Nx (tức n

nguyên dương, n ≥ x, σ(n) ≤ 2n), từ σ(n) ≤ 2n, ta có ≤ σ(n)

2n = 2n

X

d|n

d =

X

d|n

d

n, (2.48)

ở X

d|n

d để tổng tất ước tự nhiên d củan Chú ý d ước n, n

d ước n, X d|n d n = X d|n n n d = X d|n

d (2.49)

Từ (2.48) (2.49), suy

1 ≤ σ(n) 2n =

1 2n

X

d|n

d =

X

d|n

1

d (2.50)

Ta có

T (x) = X

n∈Nx

1 ≤ X

n∈Nx X d|n d ≤ [x] X n=1 X d|n d = [x] X n=1 X d|n d = [x] X d=1 X n≤x

n d

1 d = [x] X d=1 d X n≤x

n d

1

(Ở ta dùng kí hiệu P

n≤x

n d

a để tổng cho số hạng a số hạn số nguyên dương n thỏa mãn n ≤ x, n d )

Hiển nhiên ta có X

n≤x

n d

1 = hx

d i

,

T (x) ≤ [x] X d=1 d hx d i ≤ [x] X d=1 d x d = x [x] X d=1 d2

Với [x] ≥ 4, tTa có

[x]

X

d=1

1

d2 = +

1 22 +

1 32 +

[x]

X

d=4

1 d2 <

49 36+ [x] X d=4

(56)

Suy với [x] ≥ ta được:

T (x) < x

61 36 =

61x 72 <

6x

Với [x] = 0, [x] = 1, [x] = 2, [x] = 3, ta có Nx = ∅ ( σ(1) = < 2.1, σ(2) =

1 + < 2.2, σ(3) = + < 2.3) Tx= <

6 7x Vì ∀x ∈R+ ta ln có Tx<

6x

Bài toán 2.21 Với số tự nhiên k > cho trước, kí hiệu Q(n) BCNN số n, n + 1, , n + k Chứng minh với số n có dạng n = r.k! − 1, với r ∈N, n ≤ ta ln có bất đẳng thức

Q(n) > Q(n + 1)

Lời giải Kí hiệu m = [n + 1, n + 2, , n + k], ( [α, β, , γ] để BCNN số α, β, , γ ) Với j = 1, 2, , k hiển nhiên, ta có n ≡ −1( (mod j)) ( n = r.k! − 1, mà r.k! j, ∀j = 1, 2, , k ) suy (n, j) = (n, n + j) =

Vì (n, n + j) = 1, ∀j = 1, 2, , k m = [n + 1, n + 2, , n + k] nên ta có (m, n) = suy

Q(n) = [n, n + 1, n + 2, , n + k] = [n, m] = nm

Ta lại cón + k + = r.k! + k k Dom (n + 1) ⇒ m (r.k!) ⇒ m k nên số m(n + k + 1) k vừa chia hết cho k, vừa chia hết cho n + k +

Ta có Q(n + 1) = [n + 1, n + 2, , n + k, n + k + 1] = [m, n + k + 1] ( m BCNN n + 1, n + 2, , +k )

Do nhận xét suy [m, n + k + 1] ≤ m(n + k + 1)

k , ta đến bất đẳng thức

Q(n + 1) ≤ m(n + k + 1)

k (2.51)

Do k ≥ 2, r ≥ 3, nên từ (2.51) ta có Q(n + 1) ≤ m(n + k + 1)

2 =

mn



1 + k + n



≤ mn



1 + k + 3(k − 1)



Suy

Q(n + 1) ≤ mn

(57)

Bài toán 2.22 Với số nguyên k ≥ 1, gọi h(k) ≥ số nguyên nhỏ cho với chia tập hợp {1, 2, , h(k)} thành k tập hợp con, tồn số nguyên a ≥ 0, y ≥ x ≥ cho a + x, a + y, a + x + y thuộc tập hợp Chứng minh h(k) = 2k

Lời giải Xét hai trường hợp:

a) Giả sử tồn số nguyên a ≥ 0, y ≥ x ≥ cho a + x, a + y, a + x + y thuộc tập hợp Ni đó, ≤ i ≤ k −

Do Ni= {i, i + k} nên Ta có

 

a + x = a + y = i a + x + y = i + k

Từ suy x = y = k ⇒ a = i − k ⇒ a ≤ (k − 1) − k ⇒ a < Điều vơ lí, khơng thể xảy trường hợp a)

b) Giả sử tồn số nguyên a ≥ 0, y ≥ x ≥ cho a + x, a + y, a + x + y thuộc tập hợp Nk Do Nk = {k} nên ta có

a + x = a + y = a + x + y (2.52) Do a ≥ 0, y ≥ x ≥ nên (*) khơng thể có Vậy trường hợp b) xảy

Như với tập hợpN = {1, 2, 3, , 2k −1}thì tồn cách chiaN thành k tập hợp mà khơng thỏa mãn tính chất nêu đầu bài.Dĩ nhiên Ω = {1, 2, 3, , j}, ∀j ≤ 2k − tình hình tương tự tập hợp N Theo định nghĩa số h(k)

h(k) > 2k − (2.53) Vì h(k) số nguyên nhỏ thỏa mãn (2.53) nên ta có

h(k) = 2k

Bài toán 2.23 Cho n số tự nhiên ≥ Gọi f (n) số tất cách biểu diễn n thành tổng lũy thừa với số mũ nguyên không âm Hai cách biểu diễn khác thứ tự số hạng xem (

(58)

a) f (n) ≤ f (n + 1); b) f (2n) ≤ nf (n); c) f (4n) ≥ 2nf (n);

d) n2

4 < f (2n) < 2

n2

2 ∀n ≥

Lời giải a) Chú ý = 20, cộng vào cách biểu diễn n ta cách biểu diễn n + 1, f (n) ≤ f (n + 1), suy điều phải chứng minh

b) Theo câu a) suy

f (2n + 1) ≥ f (2n) (2.54) Xét cách biểu diễn số 2n + Vì 2n + lẻ nên cách biểu diễn số hạng phải xuất số lẻ lần ( ý 2k với k ≥ số chẵn) Nếu bỏ số hạng 1, ta có cách biểu diễn số 2n Như ứng với cách biểu diễn số 2n + 1, có cách biểu diễn số 2n Nói cách khác f (2n + 1) ≤ f (2n) Kết hợp với (2.54) ta có

f (2n + 1) = f (2n)

Từ cách lập luận tương tự suy lập luận sau:

i) f (2n + 1) = f (2n) = f (2n − 1) + f (n) = f (2n − 2) + f (n), ∀n ≥ ii) f (2n) = + f (2) + f (3) + f (4) + + f (n), ∀n ≥

Bây ta chứng minh bất đẳng thức câu b)

Theo ii) ta có

f (2n) = + f (2) + f (3) + f (4) + + f (n) (2.55) Theo kết câu a)

(59)

f (n − 1) ≤ f (n)

Kết hợp với (2.55) suy với mọin ≥ 2, f (2n) ≤ nf (n) Đó đpcm Dễ thấy dấu đẳng thức xảy n = n =

c) Xét bất đẳng thức f (4n) ≥ 2nf (n)∀n ≥ Ta chứng minh bất đẳng thức sau

Khi n = 2, ta có f (4n) = f (8) Dựa vào i) ta có f (1) = 1;

f (3) = f (2) = 2;

f (5) = f (4) = + f (2) = 4; f (7) = f (6) = f (4) + f (3) = 6; f (9) = f (8) = f (6) + f (4) = 10

Từ suy n = f (4n) = 10 Lại có 2nf (n) = 4f (2) = Vì 10 > nên bất đẳng thức cần chứng minh với n =

Bây xét n ≥ ta có nhận xét sau Với ≤ k ≤ n − 2,

f (n − k) + f (n + k + 1) ≤ f (n − k − 1) + f (n + k + 2) (2.56) Chứng minh (2.56) sau: Xét hai trường hợp sau

i) Nếu n − k = 2a + với a ≥ Khi từ i) ta có

f (n − k) = f (2a + 1) = f (2a) = f (n − k − 1), (2.57)

f (n + k + 1) = f (2n + 2k + 2) = f (2a + 2k + 3) = f (n + k + 2) (2.58) Từ (2.57) (2.58) suy (2.56) có dấu xảy

ii) Nếu n − k = 2a với a ≥ Lại theo câu b) ta có

f (n + k + 2) − f (n + k + 1) = f (2a + 2k + 2) − f (2a + 2k + 1) (2.59) = f (2a + 2k + 2) − f (2a + 2k)

(60)

f (n − k) − f (n − k − 1) = f (2a) − f (2a − 1) (2.60) Lại thấy

f (2a) − f (2a − 1) =  

f (a) với a ≥

f (2) − f (1) với a =

Để ý f (2) − f (1) = − = = f (1) = f (a) ( a = ) Như với a ≥ f (2a) − f (2a − 1) = f (a) Do từ (refeq18.9) suy

f (n − k) − f (n − k − 1) = f (a)∀a ≥ (2.61) Áp dụng câu a) ta có

f (a + k + 1) ≥ f (a) Vì từ (2.59) (2.61) ta có

f (n + k + 2) − f (n + k + 1) ≥ f (n − k) − f (n − k − 1), suy

f (n + k + 2) + f (n − k − 1) ≥ f (n − k) + f (n + k + 1) Vậy (2.56) Nhận xét chứng minh

Trong (2.56) thay k từ đến n − ta có

2f (n) ≤ f (n) + f (n + 1) ≤ f (n − 1) + f (n + 2) ≤ f (n − 2) + f (n + 3) ≤ (2.62) ≤ f (1) + f (2n)

Mặt khác theo (2.55) ta có

f (4n) = + f (2) + f (3) + + f (2n − 1) + f (2n) = + f (2n) +

n−2

X

k=0

[f (n − k) + f (n + k + 1)]

Để ý

n−2

X

k=0

[f (n − k) + f (n + k + 1)] ≥ 2(n − 1)f (n) Từ suy

(61)

Theo phần a) ta có

f (2n) ≥ f (2n − 1) ⇒ + f (2n) ≥ f (2) + f (2n − 1)( = f (2)) (2.64) Lại có

f (2) + f (2n − 1) ≥ 2f (n)( theo (2.64)) Vì từ (2.63) (2.64) đến bất đẳng thức

f (4n) ≥ 2nf (n) d) Xét bất đẳng thức

f (2n) < n2

2 với n ≥ (2.65)

Khi n = 3, ta có f (23) = f (8) = 10 Mặt khác, ta có

2

2 > 10 ⇒ f (23) < 32

2 Vậy bất đẳng thức (2.65) n =

Giả sử (2.65) n = k(k ≥ 3), tức ta có

f (2k) < k2

2 (2.66)

Xét n = k + Áp dụng câu b) ta có

f (2k+1) = f (2.2k) ≤ 2kf (2k) (2.67) Từ (2.66) (2.67) suy

f (2k+1) ≤ 2k.2 k2

2 = < Vậy (2.65) n = k +

Theo nguyên lí quy nạp, suy (2.65) với n ≥ Bây xét bất đẳng thức

f (2n) > n2

(62)

Khi n = 3, ta có f (23) = f (8) = 10,

4 = 4.214 <

2 < 10 Vậy (2.68) n = Giả sử (2.68) đến n = k(k ≥ 3), tức

f (2k) > k2

4 (2.69)

Xét n = k + 1, ta có

f (2k+1) = f (4.2k−1) Theo câu c) suy

f (4.2k−1) ≥ 2.2k−1f (2k−1) = 2kf (2k−1) Từ theo giả thiết quy nạp (2.69) suy

f (4.2k−1) > 2k2(k−1)24 =

4k+(k−1)2 =

(k+1)2

Do f (2k+1) >

(k+1)2

Vậy (2.68) n = k + Theo nguyên lý quy nạp suy ∀n ≥ có f (2n) >

n2

Bài toán giải

hồn tồn

Bài tốn 2.24 Cho k1, k2, , kn số nguyên dương Đặt

k = k1+ k2+ + kn

Chứng minh bất đẳng thức sau

k1!k2! kn! ≥

 k n

 !

n ,

ở dây [α] dùng để phần nguyên số α

Lời giải Khơng giảm tổng qt, ta giả sử k1 ≥ k2 ≥ ≥ kn Gọi p

số nguyên cho kp ≥

 k n



, kp+1<

 k n 

Chỉ có khả xảy

i) Nếu p = n

k1 ≥ k2 ≥ ≥ kn ≥

 k n 

⇒ k1!k2! kn! ≥

 k n

 !

n

(63)

ii) Nếu p = 

k n



> k1 ≥ k2≥ ≥ kn ⇒ k = k1+ k2+ + kn < n

 k n



≤ n.k

n = k ⇒ k < k Điều vô lý này, chứng tỏ p >

iii) Nếu < p < n Từ k1 ≥ k2 ≥ ≥ kp ≥

 k n 

, nên tồn số nguyên

li ≥ 0, ∀i = 1, p, cho

ki=

 k n 

+ li, ∀i = 1, p (2.70)

Tương tự cho 

k n



> kp+1 ≥ kp+2 ≥ ≥ kn, nên tồn số nguyên mj >

0(mj > 0, j = p + 1, n) cho

kj =

 k n



− mj, j = p + 1, n (2.71)

Mặt khác, n 

k n



≤ n, k

n = k, nên từ (2.70) (2.71) ta có

n  k n  = p X i=1

(ki− li) + n

X

j=p+1

(kj+ mj) ≤ k ⇒ k = p

X

i=1

ki+ n

X

j=p+1

kj− p

X

i=1

li− n X j=p+1 mj (2.72) ⇒ p X i=1

li= n

X

j=p+1

mj

Với i = 1, p, theo (2.70) ta có (ki)! =

 k n

 + li

 ! =  k n  !  k n  +   k n  +   k n  + li

 =  k n  !  k n 

− mj+

  k n



− mj+

  k n  −   k n 

Vậy k1!k2! kn! =

 k n  ! n A

B; A tích gồm có

p

X

i=1

li thừa số, mà

mỗi thừa số ≤ 

k n



!; cịn B tích gồm

n

X

j=m+1

(64)

đều ≤ 

k n



Mà theo (2.72) suy A

B ≥ Do

k1!k2! kn! ≥

 k n

 !

(65)

Chương 3.

Một số dạng toán liên quan

Chương trình bày số tốn cực trị đề thi học sinh giỏi

quốc gia, Olympic khu vực quốc tế liên quan đến bất đẳng thức số học

3.1 Các dạng toán bất đẳng thức số học qua các kỳ Olympic

Bài tốn 3.1 (Cộng hịa Dân chủ Đức 1974)

a) Chứng minh không tồn đa thức P (x) để với x ∈R có bất đẳng thức (3.1) : P0(x) > P00(x) (3.2) : P (x) > P00(x)

b) Khẳng định không thay đổi bất đẳng thức (3.1)

bất đẳng thức (3.3) : P (x) > P0(x) ?

Lời giải

a) Nếu P(x) số P0(x) = P00(x) = 0, bất đẳng thức (3.1) không thỏa mãn Giả sử bậc P(x) n với n ≥ 1,

Nếu n lẻ deg (P (x) − P00(x)) = n số lẻ, từ P (x) − P00(x) ≤ với điểm x ∈R.

Nếu n chẵn deg (P0(x) − P00(x)) = n − số lẻ, từ P0(x) − P00(x) ≤ với điểm x ∈R.

Như đa thức P(x) không thỏa mãn bất đẳng thức (3.1)

(66)

b) Chọn đa thức P (x) = x2+ với x ∈R, ta có P (x) − P0(x) ≡ x2− 2x + > P (x) − P00(x) ≡ x2+ > 0, nghĩa khẳng định khơng cịn

Bài tốn 3.2 (VMO - 95) Hãy xác định tất đa thức P(x) thỏa mãn điều

kiện sau:

Với số a > 1995 số nghiệm thực phương trình P (x) = a (mỗi nghiệm tính với số bội nó) bậc đa thức P(x), nghiệm

thực phương trình lớn 1995

Lời giải Do yêu cầu nghiệm thực P(x) = a lớn 1995 nên

xét đa thức P(x) có bậc n ≥

- Xét đa thức P(x) bậc n hàm đơn điệu (−∞; +∞) thỏa mãn đề Vì đồ thị hàm P(x) có hữu hạn điểm uốn nên với a đủ lớn a > 1995 P (x) = a có tối đa nghiệm (mỗi nghiệm tính với số bội nó), suy n = P(x) có dạng bx + c với b > 0; nghiệm P(x) làx = a − c

b Ta có x > 1995 với a > 1995 b > c ≤ 1995(1 − b)

- Xét đa thức P(x) có hàm số cực trị (−∞; +∞) thỏa mãn đề n ≥ Giả sử P(x) đạt cực đại m điểm u1; u2; ; um(m ≥ 1) đạt cực tiểu

tại k điểm v1; v2; ; vk(k ≥ 1)

Đặt d = max {P (u1); P (u2); ; P (um); P (v1); P (v2); ; P (vk)}

Do đồ thị hàm P(x) có hữu hạn điểm uốn nên với a đủ lớn a > max {d, 1995},

P(x) = a có tối đa hai nghiệm (mỗi nghiệm tính với số bội nó),

suy n =

Nhưng P(x) tam thức bậc hai với a đủ lớn a > 1995 P(x) = a

chỉ có tối đa nghiệm lớn 1995, đa thức lại khơng thỏa mãn đề

Vậy đa thức P(x) thỏa mãn đề có dạng P(x) = bx + c với b > c ≤ 1995(1 − b)

Bài toán 3.3 (TST 1994) Cho p(x) ∈R[x] deg p(x) = Giả sử p(x) = có nghiệm dương phân biệt Chứng minh − 4x

x2 p(x) + 

1 − − 4x x2



(67)

p00(x) = có nghiệm dương phân biệt

Lời giải Ta chứng minh bổ đề sau p(x) = có nghiệm phân biệt

0 < x1< x2< x3 < x4 phương trìnhp(x) − p0(x) = 0có nghiệm phân biệt

y1, y2, y3, y4 thỏa mãn < x1< y1 < x2< y2< x3< y3< x4 < y4

Xét f (x) = e−x.p(x) f (x) = ⇔ p(x) = Vậy f (x) = có nghiệm < x1 < x2 < x3< x4

Áp dụng định lý Lagrange ta có f0(x) = −e−x.p(x) + p0(x)e−x f0(x) = ⇔ p(x) − p0(x) =

Do phương trình p(x) − p0(x) = có nghiệm y1 < y2 < y3 deg(p − p0) =

nên phương trình p(x) − p0(x) = có nghiệm thứ y4 Khơng giảm tổng qt

ta giả sử hệ số x4 p(x) dương Suy lim

x→∞ (p(x) − p

0(x)) = +∞ nên ∃α > x4 sao cho p(α) − p0(α) > 0.

Do p(y3) − p0(y3) = p(y2) − p0(y2) = nên y4 ∈ (β, α)

Vậy < x1< y1 < x2< y2< x3< y3< x4< y4

Suy Q(x) := p(x) − p0(x) = có nghiệm dương phân biệt y1, y2, y3, y4

Giả sử Q(x) = ax4+ bx3+ cx2+ dx + e (a, e 6= 0) Khi R(x) := x4Q

1 x



= có nghiệm dương phân biệt y1 , y2 , y3 , y4 , Vậy nên R(x) − R0(x) = có nghiệm dương phân biệt

Ta có R(x) − R0(x) = ⇔ x4Q

1 x



− 4x3Q 1

x 

+ x2Q0 1

x 

= ⇔ x4hp1

x 

− p01 x

i

− 4x3hp1

x 

− p01 x

i

+ x2hp01 x



− p001 x

i = ⇔ (x4− 4x3)p1

x 

+ (−x4+ 4x3+ x2)p0 1

x 

− x2p001

x 

= ⇔ (x2− 4x)p1

x 

+ (−x2+ 4x + 1)p0 1

x 

− p001 x

 = Đặt t =

x, phương trình có dạng

1 − 4t t p(t) +



1 − − 4t t2



p0(t) − p00(t) = 0, toán chứng minh

Bài toán 3.4 (VMO - 2012) Cho hai cấp số cộng(an) , (bn) với m số nguyên

dương, m > Xét m tam thức bậc hai pk(x) = x2+ akx + bk (k = 1, 2, , m)

Chứng minh p1(x) pm(x) khơng có nghiệm thực tam thức

(68)

Lời giải Ta có tam thức bậc haip1(x)và pm(x) khơng có nghiệm thực suy

p1(x) > ∀x ∈R pm(x) > ∀x ∈R

Giả sử tồn pk(x) = x2+ akx + bk (k = 2, , m − 1) có nghiệm thực x = c Khi pm(x) − pk(x) = (m − k) (ax + b)

pk(x) − p1(x) = (k − 1) (ax + b)

(ở a, b công sai hai cấp số cộng(an) , (bn))

Do pm(c) = (m − k) (ac + b) p1(c) = − (k − 1) (ac + b)

nên pm(c) p1(c) < 0, vô lý

Vậy tam thức bậc hai cịn lại khơng có nghiệm thực

Bài toán 3.5 (IMO Shortlist 2006) Cho P (x) đa thức với hệ số nguyên có bậc n > k số nguyên dương

Xét đa thức Q (x) = Pk(x), với P (x) tác động k lần Chứng minh có nhiều n số nguyên t cho Q (t) = t

Lời giải Trước hết ta chứng minh bổ đề: Nếutlà số nguyên thỏa mãnQ (t) = t P2(t) = t

Thật ta có

(P (t) − t)|(P2(t) − P (t))| |(Pk(t) − Pk−1(t))|(Pk+1(t) − Pk(t)) Mặt khác,

Pk+1(t) − Pk(t) = P (t) − t nên

|P (t) − t| = P2(t) − P (t)

= · · · =

Pk(t) − Pk−1(t) , Đặt d = P (t) − t Nếu d = P (t) = t, suy P2(t) = P (t) = t

Nếu d 6= 0, giả sử i số nhỏ mà d = −Pi(t) − Pi−1(t), ≤ i ≤ k Khi

Pi−1(t) − Pi−2(t) = Pi−1(t) − Pi(t) Suy Pi(t) = Pi−2(t) nên P2(t) = t

Ngược lại

(69)

thì Pk(t) = t + kd 6= t, mâu thuẫn

Trở lại tốn, giả sử có (n + 1) số nguyên t1 < t2 < · · · < tn < tn+1

thỏa mãn Q (ti) = ti, ≤ i ≤ n + Khi theo bổ đề nêu trên, ta có P2(ti) =

ti, ≤ i ≤ n +

Với ≤ i < j ≤ n + 1, ta có

ti− tj|P (ti) − P (tj)

... ? ?Bất đẳng thức số học số dạng tốn liên quan? ??

đã trình bày vấn đề sau:

1 Luận văn trình bày chi tiết tính chất hàm số học

2 Tiếp theo, trình bày số dạng tốn bất đẳng thức. ..

3.2 Các đề toán toán rời rạc liên quan< /h3>

3.2.1 Một số toán cực trị tập số nguyên

Bài toán 3.6 Cho x, y, z số nguyên dương thỏa mãn điều...

[1] Phan Huy Khải (2005), Bất đẳng thức số học, NXBGD [2] Hà Huy Khoái (2004), Số học, NXBGD

[3] Nguyễn Văn Mậu (Chủ biên) (2004), Chuyên đề số học chọn lọc, NXBGD

[4] Nguyễn

Ngày đăng: 29/12/2020, 16:58

Tài liệu cùng người dùng

Tài liệu liên quan